<<

FNB MOCK TEST 2 Answers & Explanations.

Q. 1 Ratio of body weight lever arm to that of abductor musculature is

A. 1:2 B. 2:1 C. 2.5: 1 D. 1:2.5

Explanation: The ratio of the length of the lever arm of the body weight to that of the abductor musculature is about 2.5 : 1. The force of the abductor muscles must approximate 2.5 times the body weight to maintain the pelvis level when standing on one leg.

Q.2 DORR classification is used to

A. Classify femoral stem B. Classify proximal femur C. Classify cemented stem D. Classify Uncemented stem

Explanation: Dorr et al. proposed a radiographic categorization of proximal femurs based on their shape and correlated those shapes with measurements of cortical thickness and canal dimensions.

Type A femurs have thick cortices on the anteroposterior view and a large posterior cortex seen on the lateral view. The narrow distal canal gives the proximal femur a pronounced funnel shape or “champagne flute” appearance. The type A femur is more commonly found in men and younger patients and permits good fixation of either cemented or cementless stems.

Type B femurs exhibit loss from the medial and posterior cortices, resulting in increased width of the intramedullary canal. The shape of the femur is not compromised, and implant fixation is not a problem.

Type C femurs have lost much of the medial and posterior cortex. The intramedullary canal diameter is very wide, particularly on the lateral radiograph. The “stovepipe”-shaped type C bone is typically found in older postmenopausal women and creates a less favorable environment for implant fixation.

Q. 3 Highest stresses in the acetabular trabecular bone is seen with

A. When a thin-walled, polyethylene acetabular component is used and when the subchondral bone has been removed B. When a thick-walled, polyethylene acetabular component is used and when the subchondral bone has been removed C. When a metal backed polyethylene acetabular component is used and when the subchondral bone has been removed D. When a thin-walled, polyethylene acetabular component is used and when the subchondral bone has not been removed

Explanation: On the pelvic side, finite analysis has indicated that with the use of a cemented polyethylene cup, peak stresses develop in the pelvic bone. A metal-backed cup with a polyethylene liner reduces the high areas of stress and distributes the stresses more evenly. Similar studies have indicated that increased peak stresses develop in the trabecular bone when the subchondral bone is removed and that decreased peak stresses develop when a metal-backed component is used.

The highest stresses in the cement and trabecular bone develop when a thin-walled, polyethylene acetabular component is used and when the subchondral bone has been removed. A thick-walled polyethylene cup of 5 mm or more, as opposed to a thin-walled polyethylene cup, tends to reduce the stresses in the trabecular bone, similar to the effect of the metal-backed cup. The preservation of subchondral bone in the acetabulum and the use of a metal-backed cup or thick walled polyethylene cup decrease the peak stress levels in the trabecular bone of the pelvis.

Q. 4 Which is not a desired feature of a cemented femoral stem

A. Microtexturing and matte finish B. Polished surface C. Macrotexturing D. Cement centralizer

Explanation: Certain design features of cemented stems have become generally accepted.

1. Metal of high modulus of elasticity: The stem should be fabricated of high strength superalloy. Most designers favor cobalt-chrome alloy because its higher modulus of elasticity may reduce stresses within the proximal cement mantle.

2. Rounded edges and collar: The cross section of the stem should have a broad medial border and preferably broader lateral border to load the proximal cement mantle in compression. Sharp edges produce local stress risers that may initiate fracture of the cement mantle and should be avoided. A collar aids in determining the depth of insertion and may diminish resorption of bone in the medial neck.

3. Surface macro texturing: Mounting evidence suggests that failure of cemented stems is initiated at the prosthesis-cement interface with debonding and subsequent cement fracture. Various types of surface macro texturing can improve the bond at this interface ( grooves and ridges).

4. Polished surface: There is concern that even with surface modifications the stem may not remain bonded to the cement. If debonding does occur, a stem with a roughened matte surface generates more debris with motion than a stem with a smooth, polished surface. Higher rates of loosening and bone resorption were found with the use of an Exeter stem with a matte surface than with an identical stem with a polished surface.

5. No PMMA coating: The practice of precoating the stem with polymethyl methacrylate (PMMA) has been associated with a higher than normal failure rate with some stem designs and has largely been abandoned.

6. Centralizer: Neutral stem placement within the canal lessens the chance of localized areas of thin cement mantle, which may become fragmented and cause loosening of the stem. Some designs have preformed PMMA centralizers that are affixed to the distal or proximal aspects, or both, of the stem before implantation to centralize the stem within the femoral canal and provide a more uniform cement mantle. The centralizers bond to the new cement and are incorporated into the cement mantle.

Q. 5 True about dual mobility hip is all except

A. It is an unconstrained tripolar design

B. Two areas of articulation share different center of motion to increase the

C. Head –neck ratio is increased to increase stability and range of motion

D. Range is increased without increasing the impingement

Explanation: A dual mobility acetabular component is an unconstrained tripolar design. The implant consists of a porous coated metal shell with a polished interior that accepts a large polyethylene ball into which a smaller metal or ceramic head is inserted. The two areas of articulation share the same motion center. The design effectively increases the head size and the head-neck ratio of the construct. Implant impingement is reduced and stability is improved without reducing the range of motion as with constrained implants. With dual mobility systems, in vitro motion preferentially occurs at the inner bearing and the outer bearing engages at the extremes of motion.

Intraprosthetic dislocations between the small head and polyethylene ball is a complication specific to dual mobility hip joints.

Tripolar prosthesis: The tripolar-style mechanism features a small inner bipolar bearing that articulates with an outer true liner. The bipolar segment is larger than the introitus (opening) of the outer liner, preventing dislocation. This is a constrained design, which consists of a bipolar component locked into an outer polyethylene liner during the manufacturing process. The opening of the liner has an embedded metallic locking ring. The bipolar component consists of a 22 mm, 28 mm, or 32 mm prosthetic head that snaps into a polyethylene shell with a polished cobalt-chrome backing. It is free to rotate, but is locked in place by a second inner retaining ring.

Indications for constrained liners include insufficient soft tissues, deficient hip abductors, neuromuscular disease, and hips with recurrent dislocation despite well-positioned implants. Constrained acetabular liners have reduced range of motion compared with conventional inserts. Consequently, they are more prone to failure because of prosthetic impingement

Both constrained acetabular devices (tripolar prosthesis, dual mobility components cannot be relied on to compensate for technical errors in implant positioning.

Dual mobility hip Tripolar hip Constraint Unconstrained design Constrained design Center of motion between two same same articulation Second polythene head Not metal backed Metal backed ROM Increased Reduced Impingement No Yes (between femoral neck and cup) Failure rate low High

Tripolar design with a constrained ring. See second head is metal backed.

Dual mobility (un constrained) design. See second head is not metal backed.

Q. 6 True about rotator interval and its structures is

A. Rotator crescent is thick bundle of fibers found at the avascular zone of the coracohumeral ligament running perpendicular to the supraspinatous fibers and spanning the insertions of the supra- and infraspinatus tendons

B. Rotator cable is the thin, crescent-shaped sheet of rotator cuff comprising the distal portions of the supraspinatus and infraspinatus insertions

C. Intraarticular part of long head of biceps is extrasynovial

D. CHL and SGHL are main stabilizers of extraarticular part of long head of biceps

Explanation: Rotator interval: It is described as a triangular-shaped space instead of a two-dimensional ‘area’, whose borders are drawn superiorly by the supraspinatus, inferiorly by the subscapularis tendon and by the joint capsule as a ceiling. The floor of this space is the articular surface of the humeral head. It is strengthened by the CHL laterally and by the superior glenohumeral ligament (SGHL) medially. The long head of the biceps tendon lies on the base of the RI space and is covered by a fibrous sheet of capsule. The coracoid process constitutes the medial edge of this triangular space.

Microscopically, the RI consists of four layers, as described by Jost et al.The first layer consists of superficial CHL fibres which originate from the coracoid process and insert into the greater and lesser tuberosities following the supraspinatus and subscapularis tendons, respectively. The second layer is a blend of the CHL and rotator cuff tendons. The third layer consists of deep CHL fibres and the fourth layer is a mesh of the SGHL and capsule.

The rotator interval capsule is the anterosuperior aspect of the glenohumeral joint capsule, which merges with the CHL and SGHL insertions medial and lateral to the bicipital groove, maintaining the biceps tendon within the groove. The CHL arises outside the glenohumeral joint from the lateral aspect of the base of the coracoid process of the scapula. It broadens to merge with the rotator interval capsule and inserts on both the lesser and greater tuberosities; in doing so, it spans the bicipital groove.

The coracohumeral ligament (CHL) arises from the lateral side of the coracoid process and orientates obliquely to the humeral head. It divides into two layers (superficial and deep), surrounds the rotator cuff and inserts into lesser and greater tuberosities. The superficial layer of the CHL covers the articular surface of the supraspinatus and infraspinatus tendons. The deeper part of the CHL is thicker and inserts into the greater tuberosity.

The rotator crescent and rotator cable are two anatomic structures closely associated with the rotator cuff that form the shoulder's "suspension bridge."

Rotator cable : It is thick bundle of fibers found at the avascular zone of the coracohumeral ligament running perpendicular to the supraspinatous fibers and spanning the insertions of the supra- and infraspinatus tendons

Rotator crescent: The rotator crescent is a term that used to describe the thin, crescent-shaped sheet of rotator cuff comprising the distal portions of the supraspinatus and infraspinatus insertions. The crescent is bounded on its proximal margin by a thick bundle of fibers that is called as rotator cable. This cable-crescent configuration is found to consistently span the insertions of supraspinatus and infraspinatus tendons.

Long head of biceps: The long biceps tendon arises from the supraglenoid tubercle and partly from the superior glenoid labrum. The tendon is intra-articular but extrasynovial, with a surrounding synovial- lined sheath that communicates with the glenohumeral joint and extends into the bicipital groove. As it passes anteriorly and laterally to the bicipital groove, the intra-articular portion of the LHBT is cradled

and stabilized by the superior glenohumeral ligament (SGHL) and coracohumeral ligament (CHL), which fuse along the lateral half of the rotator interval with fibers from the subscapularis and supraspinatus to form the biceps pulley.

Stbilizers of long head of biceps: The structures that contribute to intra-articular stability of the long biceps tendon include the glenoid labrum and capsuloligamentous structures of the glenohumeral joint, the structures of the rotator interval including the coracohumeral and superior glenohumeral ligaments, and the supraspinatus and subscapularis tendons.

Extra-articular structures that contribute to LHBT stability include the transverse humeral ligament, the pectoralis major muscle and tendon, and the bicipital groove.

Q. 7 Which will be the most effective way for sterilization of arthroscopic equipment?

A. Autoclaving B. Ethylene oxide treatment C. Plasma sterilization D. Dipping for 20 min in Formaldehyde

Explanation: Plasmas have proven to be one of the most promising alternatives of ETO sterilization and autoclaving. Plasma sterilization system (Commercial name Sterrad-100 ) use vaporized H2O2 and radiofrequency waves for killing microbes. The sterilization cycle of Sterrad-100 first comprises a 45 min. period during which vaporized hydrogen peroxide (H2O2) is diffused throughout the treatment chamber. Then, 300 watts of radio-frequency (RF, 13.56 MHz) power is applied at a pressure of 0.5 Torr (67 Pa) to create the plasma between the perforated electrode and the grounded chamber wall; active plasma species diffuse into the chamber through the perforations.

In another commercially available system In Plazlyte, peracetic acid (PAA) vapor (a very powerful disinfecting agent) is left to diffuse, then pumped away. This is followed by microwave (MW, 2.45 GHz) plasma excitation of a low pressure gas mixture comprising oxygen, hydrogen and argon.

While high temperature autoclaving is not suitable for delicate instruments, ETO cycles comprise of 12 hours duration so can’t be used if more than one surgeries are scheduled on same day.

Q. 8 Which structure is at risk from placement of Hohmann retractor along the anterior rim of the acetabulum?

A. Femoral artery and nerve

B. Psoas tendon

C. External iliac artery and vein

D. Obturator artery

Explanation: Placement of a curved cobra or Hohmann retractor in the interval between the anterior rim of the acetabulum and the psoas tendon put the femoral nerve and vessels at risk. Erroneous placement of this retractor over the psoas muscle can cause injury to the femoral nerve or adjacent vessels. The risk increases with a more inferior placement of the retractor. The safest position is near the level of the anterosuperior iliac spine.

Q. 9 Not true about Otto Pelvis is

A. Bilateral

B. Protrusion acetabuli

C. Osteomalacia

D. Common in younger women

Explanation: Protrusio acetabuli can be primary or secondary. The primary form, arthrokatadysis (Otto pelvis), involves both hips, occurs most often in younger women, and causes pain and limitation of motion at a relatively early age (Fig. 3-76). The secondary form can be caused by migration of an endoprosthesis, septic arthritis, or prior acetabular fracture. It can be present bilaterally in Paget disease, arachnodactyly (Marfan syndrome), rheumatoid arthritis, ankylosing spondylitis, and osteomalacia. The radiographic hallmark of protrusio acetabuli is the medial migration of the femoral head beyond the ilioischial (Köhler) line.

Q. 10 years old toddler is brought to the OPD by his parents for bowed legs. He has moderate to severe varus with acute bowing below knee. The child is otherwise healthy, and height and weight are near the 50th percentiles. X ray reveals tibial metaphyseal- diaphyseal angles of 14 degree on the right and 12 degree on the left. What is the diagnosis and treatment?

A. Physiological genu varum, observation

B. Blounts disease, mermaids splints C. Pathological genu varum, valgus D. Rickets, Vit D Injection

Explanation: Physiological varus and blounts disease are close differentials. MD angle is an important measurement to differentiate between two.

From birth to 1 year a physiological (natural) varum (up to 10–15°) is present at knee. After 1 year of age, varum starts to reduce. By 18–24 months, a neutral relationship is attained. After 2 years a rapid transition to valgum occurs that peaks at 3–4 years of age (may go up to 15–20°). Thereafter, the valgum gradually reduces such that by 5–6 years of age, valgus of about 5–6° is established, which persists into adult life. At birth 5 degree of internal tibial torsion is present which gets corrected to neutral by 4-5 years of age. The tibia then gradually becomes externally rotated and reaches the adult value of 20-25 degrees of external rotation by the age of 8 years.

Q.11 Snapping hip syndrome is common in

A. Gymnast

B. Long distance runner

C. Cyclist

D. Dancers

Explanation: Snapping hip syndrome: Snapping hip syndrome (coxa saltans/dancer’s hip) is a rare disease and most commonly seen in ballet dancers and athletes requiring repetitive and extreme hip flexion and abduction. It can be intrarticular or extraarticular. Extraarticular type is more common and further divided into internal and external forms. In the internal type patients complains of pain and audible clicking sounds anterior to hip joint due to impingement of iliopsoas muscle or its tendon against iliopectineal eminence.

External form is more common and patient complains of lateral pain and clicking sounds due to rubbing of iliotibial band (ITB) over the greater trochanter. Intraarticular snapping occurs due to lesions inside the joint, viz. labral tears and loose bodies. Diagnosis is mainly clinical but dynamic USG or MRI may be helpful in clinching the diagnosis of intrarticular snapping. Tretament is mostly conservative and involve NSAIDS, ice application and stretching of involved muscle or tendon. Athroscopic loose body removal and labral repair is the treatment of choice in intrarticular snapping. Some cases of extraarticular snapping may require local corticosteroid injection into involved tedon sheath.Resistant cases may require surgical release and lengthening of iliopsoas tendon and ITB.

Q. 12 A young cricket player comes to you with knee pain and inability to extend it fully. He gives a history of twisting injury to his right knee when another player struck to his fixed leg with flexed knee. His MRI shows a double PCL sign. What is the appropriate management?

A. Arthroscopic repair B. Arthroscopic partial menisectomy C. Icing, rest, compression and elevation for 2 weeks and then isometric and knee bending exercises D. Icing, rest, compression and elevation for 2 weeks and then gradual return to sports

Explanation: Double PCL sign on MRI and inability to fully extend the knee suggest a locked knee due to entrapped bucker handle meniscal tear. Treatment acute bucket handle meniscal tear in young athletes is arthroscopic repair.

Q. 13 Sciatic nerve exploration is not indicated in which of surgery?

A. Revision THR B. THR in protrusion acetabuli C. THR in DDH D. THR in coxa vara with arthritis

Explanation: Risk of nerve palsy after primary total hip (THA) for arthritis has been found to be 0.5%, for hip dysplasia 2.3%, and 3.5% for revision surgery. Risk factors associated with nerve palsy after total hip arthroplasty include hip dysplasia, posttraumatic arthritis, posterior approach, cementless fixation, revision surgery, female sex, and significant lengthening of the extremity.

Exposure of the sciatic nerve during a posterior approach is not necessary routinely but is advisable if the anatomy of the hip is distorted, as in external rotational deformity of the femur, protrusio acetabuli, shortening of the femoral head and neck, revision surgery, or congenital dysplasia.

Q. 14 Next day after cemented total hip arthroplasty physiotherapist notes weakness of SLR and difficulty in active leg extension. Which maneuver during surgery can not cause it?

A. Lengthening more than 3 centimeters

B. Anterior capsulotomy

C. Vigorous retraction of tissues by anterior retractor

D. Extruded cement in pelvic cavity

Explanation: Injury to the femoral nerve is less common and is easily overlooked in the early postoperative period, diagnosis often is delayed. The femoral nerve lies near the anterior capsule of the joint and is separated from it only by the iliopsoas muscle and tendon. It can be injured by retractors placed anterior to the iliopsoas, during anterior capsulectomy, or with femoral retraction for acetabular preparation. The femoral nerve also can be compressed by extruded cement if acetabular cement pressurization is used. Hematoma within the iliacus muscle is another known cause of femoral nerve compression and potential palsy.

Limb lengthening has been linked to peroneal branch of sciatic nerve and complete sciatic nerve palsy.

Q. 15 Which is an indication for

A. Young patient with hip arthritis with oteopenia

B. Active sports man younger than 60 years with osteoarthritis hip joint

C. Young patient with AVN with More than 30% head involvement

D. Posttraumatic malunited trochanter hip arthritis in active patients

Explanation: Indications for hip resurfacing

1. Active men younger than the age of 60 years with osteoarthritis or posttraumatic arthritis.

2. Male manual laborers with osteoarthritis hip joint and who are required to squat

Prerequisite:

1. Nearly normal proximal femoral anatomy 2. Satisfactory bony substrate for implant fixation. As a rule, three fourths of the femoral head should be intact after contouring the head.

Contraindications:

1. Patients with osteonecrosis(more than 20-30% head involvement) or large femoral head cysts

Patients with osteoporosis should not be considered candidates for this type of procedure.(risk of fracture neck of femur)

Q. 16 Ideal position for hip is

A. 20 to 30 degrees of flexion, 0 to 5 degrees of adduction, and 0 to 15 degrees of external rotation B. 10 degrees of flexion, 0 to 5 degrees of adduction, and 20 degrees of external rotation C. 10 degrees of flexion, 0 to 5 degrees of abduction, and 0 to 15 degrees of external rotation D. 20 to 30 degrees of flexion, 10 degrees of abduction, and 0 to 15 degrees of external rotation

Explanation: Regardless of the technique selected, the ideal fusion position is 20 to 30 degrees of flexion, 0 to 5 degrees of adduction, and 0 to 15 degrees of external rotation.

Q 17 Tonnis grading is used for

A. Arthritis grading for hip preservation surgery

B. Hip infection grading

C. Grading of femoroacetabular impingement

D. Hip resurfacing

Explanation:

The Tönnis grading system is commonly used to describe the presence of osteoarthritis in hips being considered for hip preservation surgery:

Grade 0: no signs of osteoarthritis

Grade 1: sclerosis of the joint with minimal joint space narrowing and osteophyte formation

Grade 2: small cysts in the femoral head or acetabulum with moderate joint space narrowing

Grade 3: advanced arthritis with large cysts in the femoral head or acetabulum, joint space obliteration, and severe deformity of the femoral head

Q. 18 All are true regarding spinal anatomy except? A. C5 nerve root exits above the C5 pedicle B. L5 nerve root exits below L5 pedicle C. Filum terminale is modification of pia matter D. Vertebral artery travels through transverse foramen of C1 to C7 vertebral body

Explanation: C1 to C7 have a transverse foramen. Vertebral artery travels through transverse foramen of C1 to C6. C7 despite having a transverse foramen, the vertebral artery does NOT travel through it in the majority of individuals.

Q. 19 All are true regarding intervertebral disc except?

A. Annulus fibrosis composed of type I collagen

B. Nucleus pulposus shows highest compressive stress

C. Intradiscal pressure is highest while standing

D. Disc aging lead to decrease in proteoglycans content

Explanation: Intradiscal pressure is position dependent. Pressure is lowest when lying supine, intermediate when standing and highest when sitting and flexed forward with weights in the hands, When one carries weight in the hands, the closer the object is to the body the lower the pressure.

Age related changes in the disc:

Disc aging leads to an overall loss of water content and conversion to fibrocartilage. Specifically there is a decrease in nutritional transport, water content, absolute number of viable cells ,proteoglycans , pH & increase in an increase keratin sulfate to chondroitin sulfate ratio

Structure:

Annulus fibrosis is outer structure that encases the nucleus pulposus composed of type I collagen that is obliquely oriented, water, and proteoglycans

Nucleus pulposus central portion of the intervertebral disc that is surrounded by the annulus fibrosis composed of type II collagen, water, and proteoglycans approximately 88% water

Stresses :

Annulus fibrosous - highest tensile stresses

Nucleus pulposus - highest compressive stress

Q. 20 Spinal shock is characterized by all except?

A. flaccid areflexic paralysis B. Absence of bulbocavernosus reflex

C. Neurological assessment can’t be done in stage of spinal shock D. Bulbocavernous reflex returns in all cases of Injury to cauda equina when spinal shock is over

Explanation:

Spinal shock is characterized by:

➢ flaccid areflexic paralysis ➢ bradycardia & hypotension (due to loss of sympathetic tone) ➢ absent bulbocavernosus reflex - reflex characterized by anal sphincter contraction in response to squeezing the glans penis or tugging on an indwelling Foley catheter

Evaluation of spinal shock is important because one cannot evaluate neurologic deficit until spinal shock phase has resolved. End of spinal shock indicated by return of the bulbocavernous reflex. Conus or cauda equina injuries may lead to permanent loss of the bulbocavernous reflex.

Q. 21 All are true regarding halo vest immobilisation except?

A. Most rigid form of cervical spine external immobilization

B. Contraindicated in patient with cranial fractures

C. Oculomotor is most commonly injured cranial nerve

D. Ideal for upper cervical injury patient

Explanation: Abducens nerve is most common to get involved in halo vest immobilization

Q. 22 Far lateral disc herniation at L5-S1 level will cause

A. Absent plantar reflex

B. Weakness of hip abduction

C. Exaggerated EHL

D.

Explanation:

Nerve Root involved in a Prolapsing Disk

While a vertebral lesion (e.g. fracture, tumor, tuberculosis) by convention, involves the corresponding nerve root (for instance, L4 root in L4 fracture), the case is not same in disk prolapse. Disk prolapse is generally seen in lower lumbar spine with most common site being L4–L5 followed by L5–S1. We already know that every spinal nerve exits the canal by passing through the intervertebral foramen present on the sides of the corresponding vertebrae, e.g. L4 nerve root will pass out of the canal by exiting through the intervertebral foramen on the side of the L4 vertebrae. Now, consider the situation when there is a disk prolapse L4–L5. The disk is present below the L4 vertebrae and if this disk goes back, it cannot compress the L4 nerve root as the same has already exited from the foramen present on the sides of the vertebrae. However, the L5 nerve root is coming from the L1 level, where the cord has already ended and thus L5 is the traversing nerve root in the canal which is traveling down to exit on the side of the L5 vertebrae. Henceforth, if the disk between L4 and L5 prolapses, it would be the L5 nerve root that would be affected. The same applies to all lumbar nerve roots and it is always the lower level nerve root that is involved by a prolapsing disk.

Caution: Even in cervical disk prolapse where the nerves actually exit from the top of the corresponding vertebrae rather than below it, the lower nerve root is involved. This is because while in lumbar spine the disk involves traversing nerve root, in cervical spine the disk involves exiting nerve root due to a varied anatomy.

Far lateral disc: However, in far lateral disc herniation in lumbar spine the same level nerve root will be involved i.e. a far lateral disc at L4-L5 would compress L4 nerve root and it will spare posteriorly situated L5 root . In cervical disk prolapse isolated nerve involvement occurs only if the disk prolapse is far lateral, as most other disks tend to indent cord present in the cervical area.

Far lateral disc at level of L5-S1 level will compress the L5 nerve root. L5 nerve innervates the EHL so best answer here would have been week EHL. This is not given in the answers so most appropriate answer here would be trendelenburg gait due to weak hip abduction. Root value of superior gluteal nerve which supplies hip abductors is L4-L5-S1.

Q. 23 35 year male presented with neck pain with left upper limb radicular pain for 4 weeks. On examination pain was relieved by elevation of shoulder above head. His MRI neck was taken and shown below. All will be related to MRI finding except?

A. Wrist flexion weakness

B. Diminished triceps reflex

C. Spurling Test positive

D. flexion weakness

Explanation:

A herniated disc at cervical segment 4 and 5 (C4-C5) causes C5 nerve root impingement. Patients may feel weakness in the deltoid muscle in the upper arm but do not usually feel numbness or tingling sensations. A cervical herniated disc at this level can also cause shoulder pain.

A herniated disc at cervical segment 5 and 6 (C5-C6) causes C6 nerve root impingement. This level is one of the most common areas for a cervical herniated disc to occur. It can cause weakness in the biceps (weak elbow flexion) and in the wrist extensor muscles. Pain, numbness and tingling can radiate to the thumb side of the hand.

C6-C7 disc herniation (case given in the question) will cause C7 radiculopathy with following

• triceps and wrist flexion weakness

• diminished triceps reflex

• paresthesia in the index, middle, ring

C8 radiculopathy is caused by herniated C7-C8 disc and gives rise to following features

• weakness to distal phalanx flexion of middle and index finger (difficulty with fine motor function)

Paresthesias in little finger

Q. 24 A 65 year old male from Punjab presented with difficulty in walking for 3 month with neck pain. He has history of neck injury 1 year back. On examination his DTR are exaggerated with positive hoffmann reflex. CT scan of neck was taken and shown below. Which is most likely diagnosis?

A. Post traumatic OPLL B. Fluorosis C. DISH D. Ankyloing spondylitis

Explanation: This is a case of ossification of the posterior longitudinal ligament (OPLL). It is a hyperostotic condition that results in ectopic calcification of the posterior longitudinal ligament. Cervical spine is most commonly involved followed by thoracic and lumbar spine. This typically seen in the mid cervical spine and results in central canal stenosis, predisposing the patient to cord injury from minor trauma. The incidence of OPLL in the Asiatic population is 2%–3%, but the incidence is more in the background of fluorosis. Incidence of OPLL in Caucasians is 0.2- 0.7%. Etiology Although the cause of OPLL has been debated, fluoride intoxication, diabetes mellitus, growth hormone imbalance, recurrent minor trauma, abnormal calcium metabolism, and infection have been suggested. Fluorosis is associated with a higher incidence of OPLL. Higher urinary fluoride levels correlate with the severe forms of OPLL. In the fluorotic zones, there is an increase in the number of cases of myelopathy due to cervical OPLL, and therefore, this entity is the cause of major concern. Apart from Fluorosis, OPLL is often associated with several other entities: -Diffuse idiopathic skeletal hyperostosis (DISH) -Ossification of the ligamentum flavum -Ankylosing spondylitis

Classification: A lateral radiograph is used to classify OPLL into four subtypes: continuous, segmental, mixed, and localized or other. Types of OPLL

• type 1: continuous (ossification of several vertebral segments)

• type 2: segmental (several segments are affected, ossification is disrupted)

• type 3: mixed (mix of types 1 and 2)

• type 4: localized (circumscribed, only one or two segments)

The mixed and continuous types are most frequently associated with progression to myelopathy. Although 5% of diagnosed patients are asymptomatic, varying degrees of neurologic symptoms can be present including both radiculopathy and myelopathy. Occupancy ratio (a/b), defined as the anterior-to-posterior ratio of the OPLL to the spinal canal, can be calculated on lateral radiographs or CT sagittal images. A ratio of 30 to 60% is predictive of the development of myelopathy.

Ossification of posterior longitudinal ligament (OPLL) is a progressive disease that causes spinal canal compromise and serious neurological squeal in advanced cases.

Q. 25 All are true regarding Pediatric Spinal Cord Injury except?

A. Cervical spine injuries more common in children aged <8 years due to small head-to body ratio.

B. RTA is most common cause C. Pediatric spinal column can stretch up to 5 cm without rupture. D. Normal retropharyngeal space is < 6 mm at C2 and < 22 mm at C6

Explanation:

Cervical spine injuries are more common in children aged <8 years due to large head to body ratio

Q. 26 The Lenke classication for adolescent idiopathic scoliosis include all except?

A. Type 1-6 Primary curve B. Lumbar modifier (A,B,C) C. Sagittal modifier (-,N,+) D. Thoracic modifier(A,B,C)

Explanation: The Lenke classication for adolescent idiopathic scoliosis has gained popularity and consists of three steps 1. Label primary curve at Type 1-6 2. Assign lumbar modifier (A,B,C) 3. Assign sagittal modifier (-,N,+)

Q. 27 The typical anterior approach to the cervical spine uses the plane between which of the following structures?

A. The carotid artery (laterally) and jugular vein (medially) B. The carotid sheath (laterally) and the trachea and oesophagus (medially) C. The inferior thyroid artery (medially) and the carotid sheath (laterally) D. The brachial plexus (laterally) and the carotid sheath (medially)

Explanation: There is no true internervous plane during the anterior approach to the cervical spine. The surgical plane lies between carotid sheath (laterally) and the trachea and oesophagus (medially).

Q. 28 Which of the following statements is false when comparing vertebroplasty with kyphoplasty?

A. In kyphoplasty one of the goals is to restore vertebral height and this is not the case in vertebroplasty B. PMMA bone cement is injected at a higher pressure in kyphoplasty when compared with vertebroplasty C. Kyphoplasty requires additional equipment compared with vertebroplasty D. Improvement in the level of pain is comparable in both procedures

Explanation:

Direct comparisons between vertebroplasty and kyphoplasty are not possible because of a lack of prospective data comparing the two approaches. Indirect comparisons suggest that vertebroplasty and kyphoplasty reduce pain comparably. Both procedures appear to improve patient function in most series, although available data usually cannot be pooled because of a wide variety of measurement scales. Vertebral height restoration and reduction of kyphotic deformity are also similar for both procedures. Cement is injected under the same pressure in both kyphoplasty and vertebroplasty. The vertebral height is restored during kyphoplasty by inflation of a balloon inserted into the vertebral body. Pain relief is comparable in both procedures.

Q. 29 Which of the following nerves is at risk while performing a surgical repair of the tendo Achillis?

A. Sural nerve B. Saphenous nerve C. Tibial nerve D. Lateral plantar nerve

Explanation:

The sural nerve passes lateral to the tendo Achilles and can be injured during repair of the tendon. Making a skin incision slightly medial to the tendo Achilles when performing a surgical repair can reduce this risk.

Q. 30 True about supracondylar fracture in children are all except

A. Obese children have a higher rate of both preoperative and postoperative nerve palsy B. Approximately 5% to 10% of children have an associated ipsilateral distal radial fracture C. Ulnar nerve injuries are common in flexion type and majority of cases recover fully within 3 months D. Compartment syndrome occurs in approximately 0.1% to 0.3% of patients with supracondylar humeral fractures.

Explanation: Facts to be remembered about supracondylar fractures in children ➢ Compartment syndrome occurs in approximately 0.1% to 0.3% of patients with supracondylar humeral fractures and is more common with concurrent fracture of the forearm or wrist fractures ➢ Long-term follow-up studies have shown that most patients had excellent long term function; 100% of patients with radial nerve, 88% of patients with median nerve, and only 25% of patients with ulnar nerve injuries fully recover. ➢ The most common age of injury is 5 to 7 years. Almost all (98%) are extension-type injuries, Flexion-type fractures, although rarer (2%), are more difficult to reduce, have worse outcomes, and are associated with ulnar nerve injury. ➢ A careful neurologic examination is essential because 10% to 15% of patients have a nerve injury, with the anterior interosseous nerve being the most frequently injured in extension-type fractures. The ulnar nerve is most frequently injured in flexion-type injuries. ➢ A vascular injury, typically to the brachial artery, can occur in up to 10% to 20% of patients with a type III fracture. Obese children with supracondylar fracture have been shown to have a higher rate of both preoperative and postoperative nerve palsy

Q. 31 A 5-year-old boy presents to A & E with a painful hip associated with a limp for the past 48 hours. He has a temperature of 38.5°C and an elevated CRP with a high white cell count. Radiographs suggest an effusion in the hip. The next best management step would involve

A. Blood cultures and administration of intravenous antibiotics B. Ultrasound-guided aspiration and intravenous antibiotics C. Intravenous antibiotics alone D. Emergency aspiration and washout of the hip joint followed by intravenous antibiotics

Explanation: Emergency aspiration and washout of the hip joint followed by intravenous antibiotics Ultrasound is the

investigation of choice when imaging a child with a painful hip.The use of diagnostic algorithms is no substitute for careful clinical examination. Microbiological cultures can be unreliable and provides the opportunity to obtain tissue for culture.

Q. 32 Which of the following factors is least predictive of a poor outcome after whiplash injury?

A. Headache B. Immediate pain C. Pain isolated to neck D. Severe pain in first week

Explanation: The term ‘whiplash’ is used to denote the mechanism of an indirect flexion-extension soft-tissue injury to the neck resulting from a rear-end motor-vehicle collision. Symptoms associated with a worse outcome are rapid onset of pain, severity of neck pain, acute hospital admission, radiation of pain to the upper limb and headache

Q. 33 With reference to ‘pulled elbow’ injury in children, which of the following statements is true?

A. Peak incidence is between 3-5 years of age B. It results from traction on the extended and supinated forearm C. Recurrence is seen in 5% of cases D. Pain is due to tearing of the lateral collateral ligament complex

Explanation:

Recurrence is seen in 5% of cases Subluxation of the head of radius from the annular ligament is a common elbow injury. It is most common amongst young children, prior to the age of five (commonly between 1-4 years of age). Peak incidence is seen between 2-3 years. It results from traction on the hand with the elbow extended and the forearm pronated. The annular ligament either tears or slips over the radial head. After reduction, the elbow joint capsule remains stretched for a few months. Studies have reported recurrence rates up to 5%. It is reduced by flexion and supination of the forearm.

Q. 34 The alpha angle on the ultrasound of an infant’s hip is defined as

A. The angle between the acetabular roof and the midline of the pelvis B. The acute angle between the lateral wall of the ilium and the bony acetabular roof C. The angle between the centre of the femoral head and the lateral wall of the ilium D. The angle of the thigh required to produce subluxation of the hip on the sonogram

Explanation: The alpha angle is used most commonly as a measurement of acetabular concavity, and it is calculated as the angle between the lateral wall of the ilium (vertical cortex of the ilium) and the roofline (acetabular roof). A normal alpha angle is 60 degrees or greater. This measurement is vital to understanding the morphology of the immature acetabulum in developmental dysplasia. The beta angle is the angle formed between the vertical cortex of the ilium and the triangular labral fibrocartilage (echogenic triangle). Normal value is 55 degrees or less. Graf classification of DDH is based on the ultrasound value of these angles.

Q. 35 Which of these values reflects the normal tibio- femoral axis for a child aged three years?

A. Varus of 7° B. Varus of 10° C. Neutral alignment D. Valgus of 10°

Explanation: During development, the tibio-femoral alignment in children changes during early years. At birth it is 10° to 15° of varus, which remodels to neutral at about 24 months of age and then becomes 10° of valgus by the age of three years. Over the next four years it gradually goes to normal alignment of 7°.

Q. 36 Which of the following statements is correct with regard to a therapeutic intra-articular facet joint injection in the lumbar region for low back pain?

A. Pain relief on two occasions after a facet joint injection is an indication for facet joint ablation B. It aims to relieve back pain during flexion of the lumbar spine C. Steroid injection in the facet joint gives good pain relief, beyond six months D. The best visualisation of the facet is on the lateral image intensifier view

Explanation:

The facet joints are paired diarthrodial articulations located between the posterior elements of the adjacent vertebrae; also known as zygo-apophyseal joints. They are formed by the articulation of the inferior articular processes of one vertebra with the superior articular processes of the vertebra below. They are typical synovial joints with the facets covered by articular , bridging synovial membrane, tough fibrous capsule and intervening layer of loose areolar tissue. Nerve supply of facet joints: The facet joints are richly innervated by the nerve fibres from the medial branch of the dorsal ramus of spinal nerves. Each facet has a dual nerve supply, from the dorsal ramus at the same level as well as from the level above. Each spinal nerve root innervates two facets; it supplies the facet joint at the level it exits, as well as the subsequent lower facet. The exceptions to this dual nerve supply are: singular nerve supply to the atlanto-occipital joint, atlanto-axial joint and C2/3 facet joint, which are innervated by C1, C2 and C3 nerves respectively. Facet joint injections are given under C arm guidance or CT guidance in prone position. Pain relief on two occasions after a facet joint injection is an indication for facet joint ablation. If facet joint injection has provided successful pain relief the diagnosis is verified and therefore facet joint ablation is indicated. However the exact number of lumbar facet joint injections required prior to facet joint ablation is disputed Lumbar medial branch (which supplies the facet joint) block has better predictive value than facet joint injection for indication of facet joint radio frequency ablation: Three published studies to date compare the predictive values of lumbar medial branch block and facet joint injections before ablative therapy. In a case-control study by Cohen et al involving 511 patients who received RFA after either IA blocks, MBBs or both blocks, the authors reported a higher success rate with MBB than with IA injections (70.3% vs 60.8%), which remained a significant outcome predictor in multivariable analysis.

Q. 37 Success rate of coccygectomy after failed conservative treatment is

A. <50% B. 80% C. Procedure has been abandoned in favour of neural block due to high complication rate D. >90%

Explanation: Causes of coccygodynia:

The most common cause of coccygodynia was direct trauma. Other causes are, childbirth, and recent lumbar spinal surgery or rectal surgery or epidural injection.

Acute coccygodynia (symptoms <2months) is managed differently from chronic coccygodynia (symptoms >2 months).

All patients with acute coccygodynia (symptoms <2 months) are prescribed rest and a nonsteroidal anti- inflammatory drug (NSAID) for a period of 8 weeks. A stool softener is recommended in patients who have a history of pain with defecation, also for 8 weeks. Adjustments in seating with U-shaped cushions can relieve pressure on the . Additional therapies include sitz baths, hot mud application, acupuncture, and chiropractic manipulation. injection with manipulation is effective in providing pain relief, but they also suggested that coccygectomy is a reasonable choice for those in whom the conservative management failed.

Chronic coccygodynia:

Favorable results are reported after corticosteroid and local anesthetic injections given on an as- required basis. They may be given alone or in combination with invasive manipulation (i.e., transrectal flexion and extension) under general anesthesia, the latter being considered more successful. The corticosteroid plus local anesthetic should either be injected into the soft tissues around the sides and tip of the coccyx (using methylprednisolone) or the sacrococcygeal (SC) junction and dorsalperiosteum of the coccyx (using triamcinolone).

If corticosteroid injections have no effect after 2 successive monthly injections, surgical coccygectomy is offered as definitive treatment. Success rate of coccygectomy has been reportedbetween 60 %to 80 %.

Q. 38 Not a true picture for fanconi syndrome

A. Increased ALP and PTH B. Decreased serum phosphate and increased serum calcium C. Increased urine Phosphate, and Potassium levels D. Decreased 25(ΟΗ)VitD blood levels and metabolic acidosis

Explanation: Adult acquired idiopathic form Fanconi’s syndrome: Biochemical abnormalities

Fanconi syndrome is associated with renal proximal tubular dysfunction. The main pathogenic feature is a generalized reabsorption defect, leading to the wasting of phosphate, amino acids, glucose, and bicarbonate in erratic degrees. Clinical indices vary from direct or indirect turbulences of the tubule system, with bone diseases frequently presenting as sequelae of the syndrome. Therefore, clinical manifestations of the disease differ and morbidity is secondary to the produced metabolic abnormalities. Most of these abnormalities, such as acidosis, calciuria, and phosphaturia, affect bone deposit and, thus, growth.

Typical biochemical findings of fanconi syndrome is:

Decreased BMD, and osteoporosis

Blood: metabolic acidosis and a decrease in Phosphate, 25 (ΟΗ)Vit D blood levels, Increased ALP and PTH level.

Urine: Increased urine Phosphate, and Potassium levels, calciurea

Q. 39 Which one of the following is the strongest factor leading to symptomatic degenerative disc disease?

A. Heavy lifting B. Vibration forces C. Tobacco exposure D. Genetic predisposition

Explanation:

Q. 40 Which one of the following statements is not true about radial tunnel syndrome?

A. Cause of lateral elbow pain B. Motor weakness C. Electrodiagnostic studies are not helpful D. Most common site of compression is arcade of Frohse

Explanation: Radial Tunnel Syndrome Radial tunnel is approximately 5 cm long space in front of elbow extending from the radiocapitellar joint to proximal margin of supinator. The term radial tunnel syndrome (RTS) indicates compression of PIN in radial tunnel by fibrofascial bands coursing superficial to the radial head, the radial recurrent artery, the fibrous edge of the extensor carpi radialis brevis (ECRB), the proximal and distal edges of the supinator. The most common point of compression is the arcade of Fröhse (free aponeurotic proximal margin of supinator, in which case the condition is specifically designated as PIN syndrome). Radial tunnel syndrome and posterior interosseous nerve (PIN) syndrome (supinator syndrome) both are cause of intractable lateral elbow pain and can present with similar symptoms. The clinical presentation includes pain 5 cm distal to the lateral epicondyle over the course of the radial nerve down the forearm. Typically, patients have pain with resisted extension of the long finger with the elbow in extension, forearm in pronation and the wrist in neutral. Both syndromes can be differentiated by the fact that patients with PIN syndrome have a loss of motor function whereas patients with RTS have only lateral forearm pain without motor involve- ment. The difference in clinical presentation may be due to differences in the degree of compression. Electrodiagnostic studies are usually not helpful in making the diagnosis and electromyography (EMG) and nerve conduction velocity (NCV) studies in patients with RTS are typically negative Nonsurgical treatment of radial tunnel syndrome includes rest, NSAID and steroid injection if associated with lateral epicondylitis. Surgical decompression provides excellent result in most of patients.

Q. 41 Which one of the following would be the most suitable test to detect a difference between the Grade of Perthes’ disease (Herring classification) in two different population groups belonging to the rural or urban areas?

A. Paired t-test B. Mann-Whitney U test C. Wilcoxon signed rank test D. Kruskal-Wallis test

Explanation:

Here we are comparing two different groups that are not paired or matched. The Herring classification system is an ordinal measurement (i.e. Grade A, B, or C, with increasing severity).

Q. 42 All of the following statements are true about the flexion tear drop fracture of the cervical spine except:

A. Commonly at the level of C5 B. Displacement of the upper column of the cervical spine is characteristic C. Sagittal body and laminar fracture are unusual D. Characteristic neurological injury is the anterior cord syndrome

Explanation:

Its name is derived from the characteristic triangle-shaped fragment that fractures from the anteroinferior corner of the vertebral body, resembles a drop of water dripping from the vertebral body. Characteristically there is posterior displacement of the upper column of the divided cervical spine. The injury is frequently associated with sagittal body and laminar fractures. The most common level is C5 and anterior cord syndrome is the characteristic neurological injury pattern

Q. 43 Which of the following statements is true for Pemberton’s pelvic osteotomy used for the treatment of developmental dysplasia of the hip (DDH)?

A. Unsuitable to be performed as a bilateral procedure in one operative session B. Requires pin fixation C. Does not alter the volume of the acetabulum D. The osteotomy hinges at the triradiate cartilage

Explanation:

Pemberton’s pelvic osteotomy reduces the acetabular volume. It is indicated for correcting residual acetabular dysplasia in six- to seven-year-olds with DDH. The advantages of the procedure are the ability to perform bilateral at the same sitting, no need for and a greater capacity for correction. The disadvantage is the alteration of the shape of the acetabulum, which requires the procedure to performed early enough in childhood to allow time for remodelling of the acetabulum to the shape of the femoral head.

Q. 44 True about hind quarter amputation is

A. The standard hemipelvectomy employs a posterior or gluteal flap and disarticulates the symphysis pubis and sacroiliac joint B. The standard hemipelvectomy employs an anterior flap and disarticulates the symphysis pubis and sacroiliac joint C. The standard hemipelvectomy employs a posterior or gluteal flap, leaves the symphysis pubis and cut through involved pubis and iliac bone or sacroiliac joint D. The standard hemipelvectomy employs a posterior or anterior flap, leaves the symphysis pubis and SI joint and cut through the pubis and iliac bone

Explanation:

Hindquarter amputation: The standard hemipelvectomy employs a posterior or gluteal flap and disarticulates the symphysis pubis and sacroiliac joint and the ipsilateral limb. An extended hemipelvectomy includes resection of adjacent musculoskeletal structures, such as the sacrum or parts of the lumbar spine.

Anterior flap hemipelvectomy is indicated for lesions of the buttock or posterior proximal thigh that cannot be adequately treated by limb-sparing methods. The larger posterior defect is covered by a quadriceps myocutaneous flap maintained by the superficial femoral vessels and may include part of the sartorius muscle.

Earlier mortality from hemipelvectomy was greater than 50%, but with more recent advances including radiation, chemotherapy, and patient optimization, mortality is less than 10%.

Q. 45 The surgical management of ankylosing spondylitis may include osteotomies for spinal flexion deformities to restore horizontal gaze. Which one of the following is true with regard to this?

A. Motion of the spine is increased B. The Smith-Petersen osteotomy will give 10° of correction per 10 mm of resection C. Cervico-thoracic osteotomy should be undertaken prior to total because of increased risk of dislocation D. Osteotomies are usually carried out in the thoracic spine to improve respiratory function

Explanation:

Cervical oteotomies in AS: Cervicodorsal kyphosis usually can be treated satisfactorily by lumbar osteotomy, which provides a compensatory lumbar lordosis and results in an erect posture. Cervical osteotomy may be indicated, however, (1) to elevate the chin from the sternum, improving the appearance, the ability to eat, and the ability to see ahead; (2) to prevent atlantoaxial and cervical subluxations and dislocations, which result from the weight of the head being carried forward by gravity; (3) to relieve tracheal and esophageal distortion, which causes dyspnea and dysphagia; and (4) to prevent irritation of the spinal cord tracts or excessive traction on the nerve roots, which causes neurologic disturbances.

The appropriate level for osteotomy is determined by the deformity and the degree of ossification of the anterior longitudinal ligament.

Lumbar osteotomies in AS: The osteotomy usually is made at the upper lumbar level because the spinal canal here is large, and the osteotomy is distal to the end of the cord. A lumbar lordosis is created to compensate for the thoracic kyphosis; motion of the spine is not increased. Osteotomy methods include resection of the spinous processes from the laminae to the pedicles, simple wedge resection of the spinous processes into the neural foramina, chevron excision of the laminae and spinous processes, and combined anterior opening wedge osteotomy after posterior resection of the spinous processes and laminae.

Smith-Petersen osteotomy: The Smith-Petersen osteotomy is an excellent option for correction of smaller degrees of spinal deformity. Bone I removed through the pars and facet joints. Approximately 10 degrees of correction can be obtained with each 10 mm of resection. The osteotomy is closed with compression or with in situ rod contouring, and bone graft is applied.

Pedicle subtraction osteotomy: It is best suited for patients who have significant sagittal imbalance of 4 cm or more and immobile or fused discs. Pedicle subtraction osteotomy is inherently safer than the Smith-Petersen osteotomy because it avoids multiple osteotomies. Typically, 30 degrees or more of correction can be obtained with a single posterior osteotomy, preferably at the level of the deformity.

Eggshell osteotomy: It requires anterior and posterior approaches and usually is reserved for severe sagittal or coronal imbalance of more than 10 cm from the midline. This is a spinal shortening procedure with anterior decancellization followed by removal of posterior elements, instrumentation, deformity correction, and fusion.

Total Hip Arthroplasty carries less risk than spinal correction – both hips should be operated on simultaneously if they have significant flexion contractures. The pulmonary disease of ankylosing spondylitis is not simply due to reduced chest expansion, but also pulmonary fibrosis which will not be corrected by surgery. Similarly, the aorta is subject to inflammatory disease processes making it more at risk during any spinal manipulation.

Q. 46 Lumbar osteotomy indicated for severe sagittal or coronal imbalance of more than 10 cms from the mid line is

A. Pedical subtraction

B. Eggshell

C. Smith-Petersen

D. This much correction is not possible by lumbar osteotomies

Explanation:

(See Q. 45)

Q. 47 False about cervical spine involvement in RA is

A. Atlanto axial instability (AAI) is more common than cranial settling

B. AAI is defined as an AADI that is greater than 3 mm and not fixed with flexion and extension as it generally increases with flexion and may reduce with extension

C. Values for PADI more that 13 or 14 mm have been suggested as indications for surgery

D. Distance between the center of the C2 pedicle and the transverse axis of C1 measured along the axis of the odontoid process more than 15 mm in male indicates cranial settling

Explanation:

After peripheral joints, cervical spine is the 2nd most common area to be involved in RA. Risk factors for developing cervical involvement are an older age at onset, more active synovitis, higher levels of C-reactive protein, rapidly progressive erosive peripheral joint disease, and early joint subluxation. Three basic types of cervical instability are present in this disease. Atlantoaxial instability is most common, affecting 19% to 70% of patients; basilar impression or atlantoaxial impaction (cranial settling) occurs in 38%; and subaxial subluxation occurs in 7% to 29%. Cranial settling is caused by C1–C2 instability, leading to upward migration and/or rotation of the C2 complex into the cranial vault. Clinical presentation: Neck pain, specifically pain at the craniocervical junction, is one of the most common presentations, in one report occurring in 69% of patients with cervical instability. Occipital headache, mastoid pain may occur due to compression of occipital and auricular nerves. Signs of

vertebrobasilar insufficiency such as tinnitus, vertigo, visual disturbance, and dysphagia can also occur and may be due to mechanical compression. X ray evaluation: The anterior atlantodental interval (AADI) is the distance from the posterior margin of the anterior arch of C1 to the anterior margin of the dens measured along the transverse axis of C1 which in normal adults is less than 3 mm. AAI is defined as an AADI that is greater than 3 mm and not fixed with flexion and extension as it generally increases with flexion and may reduce with extension. Various cutoffs between 6 and 10 mm for maximum AADI have been suggested as indications for surgery. Posterior atlantodental interval (PADI) has been found to be a more reliable indicator of the potential for neurologic compromise. This value is obtained by measuring from the posterior margin of the dens to the anterior margin of the posterior arch of C1. Values for PADI less that 13 or 14 mm have been suggested as indications for surgery. The presence of cranial settling (CS) is best evaluated using a combination of the Clark station, Ranawat criterion, and the Redlund-Johnell criterion Indices for cranial settling:

Q. 48 Not a contraindication for cervical spine disc replacement

A. 2 level involvement B. Morbid obesity C. RA D. Cervical instability

Explanation:

Indications for cervical disc replacement: ➢ Symptomatic cervical disc disease at one or two vertebral levels between C3-T1 confirmed by imaging (MRI, CT, or myelography) showing herniated nucleus pulposus, spondylosis, or loss of disc height ➢ Failed ≥ 6 weeks of conservative therapy ➢ Between 20 and 70 years of age

➢ No contraindications Contraindications ➢ ≥3 vertebral levels requiring treatment mCervical instability (translation > 3 mm and/or > 11-degree rotational difference to that or either adjacent level) ➢ Known allergy to implant materials (titanium, polyethylene, cobalt, chromium, and molybdenum) ➢ Cervical fusion adjacent to the level to be treated ➢ Posttraumatic vertebral body deficiency/deformity ➢ Facet joint degeneration ➢ Neck or arm pain of unknown etiology, Axial neck pain as the solitary presenting symptom ➢ Severe spondylosis (bridging osteophytes, disc height loss > 50%, and absence of motion < 2 degrees) ➢ Osteoporosis/osteopenia ➢ Prior surgery at the level to be treated ➢ Active malignancy; history of invasive malignancy, unless treated and asymptomatic for at least 5 years ➢ Systemic disease (acquired immune deficiency syndrome, human immunodeficiency virus, hepatitis B or C, and insulin-dependent diabetes) ➢ Other metabolic bone disease (i.e., Paget disease and osteomalacia) ➢ Morbid obesity (body mass index [BMI] > 40 or weight > 100 lb over ideal body weight) ➢ Pregnant or trying to become pregnant in next 3 years ➢ Active local/systemic infection ➢ Presently on medications that can interfere with bone/ soft-tissue healing (i.e., corticosteroids)

Autoimmune spondyloarthropathies (rheumatoid arthritis

Q. 49 Which one of the following structures is at increased risk during a RIGHT-sided anterior approach to the cervical spine when compared with a LEFT-sided approach?

A. Recurrent laryngeal nerve B. Superior laryngeal nerve C. Carotid sheath D. Superior thyroid artery

Explanation:

The left recurrent laryngeal nerve ascends in the neck between the trachea and the oesophagus after branching from the vagus nerve and curving around the arch of the aorta. The right

recurrent laryngeal nerve runs alongside the trachea after hooking around the right subclavian artery (more proximal than the aortic arch). In the lower part of the neck it crosses from lateral to medial, to the trachea. It is therefore more vulnerable than the left recurrent laryngeal nerve.The right recurrent laryngeal nerve may also be aberrant, leaving the carotid sheath at a higher level, crossing the operative field at the level of the thyroid gland. Despite this, recurrent laryngeal nerve injury has not been shown to have an increased incidence of damage with right dissection versus left, and the overall rate is 2.7%.

Q. 50 A patient undergoes cervical spine surgery via the anterior approach. A cerebrospinal fluid (CSF) leak is diagnosed. Which of the following statements is correct?

A. CSF leak is a common complication of anterior cervical surgery B. The CSF leak is usually easily accessible C. Direct repair should not be attempted D. A lumbar CSF drain may need to be placed

Explanation:

The treatment depends upon when the CSF leak is noted (intra-operatively or post-operatively). Dural tear is common in spinal surgery as a whole, with an incidence of around 7% in primary cases (in all spinal surgery). Hannallah et al reported a rate of around 1% in a series of 2216 approaches to the cervical spine (72% of the approaches were anterior). The prevalence of cerebrospinal fluid leaks was higher in patients with a diagnosis of ossification of the posterior longitudinal ligament (12.5%), patients having a revision anterior procedure (1.92%), men (1.56%), and patients undergoing an anterior cervical and arthrodesis (1.77%). There were no long-term sequel of the cervical dural tears in the series. Dural tears recognised intra-operatively are best treated with watertight closure of the dura and soft tissues. When diagnosed in the post- operative period, the patient can initially be treated with strict bed rest. If drainage persists, a percutaneous lumbar subarachnoid catheter can be inserted to divert CSF into a closed sterile drainage system to allow healing of the dura. If this approach fails, open surgical repair is required.

Q. 51 Which of the following treatment options for scoliosis relates to surgical management via a posterior approach?

A. Boston brace

B. Cotrel-Dubousset system C. Zielke procedure D. Dwyer procedure

Explanation:

This posterior instrumentation system comprises of 7 mm rods, a series of pedicular and laminar hooks that can be secured to the rod at any point along its length and in any transverse axis of rotation, and transverse approximators. Pedicle screws can also be used. Post- operatively no bracing is needed for adolescents.

Dwyer, et al. used anterior instrumentation. They believed that scoliotic deformities could be approached in two ways, either by stretching the concave side of the curved column or by shortening the long convex side. Their patients wore a jacket to restrict movement for up to three months postoperatively. The Zielke technique utilises ventral convex-side compression to achieve scoliosis correction. This resulted in greater correction of deformities than previous techniques. This method improved on the Dwyer technique by requiring that the VB screws be placed more posteriorly to enhance the derotation effect and reduce the incidence of iatrogenic kyphosis. Although excellent correction of deformity was possible via the anterior approach, significant incidences of pseudoarthrosis, hardware failure, and loss of correction with the single rod technique were reported. Dual rod anterior instrumentation techniques were introduced to improve upon this

Q. 52 A patient is having a trans-articular C1/C2 screw fixation. Which structure is most at risk?

A. C2 nerve root B. Hypoglossal nerve C. Carotid artery D. Vertebral artery

Explanation:

The indications for C1-C2 transarticular screw fixation include traumatic injuries to the atlantoaxial complex, instability resulting from inflammatory disease (rheumatoid arthritis), and congenital abnormalities (os odontoideum). The structure that is most at risk is the vertebral artery in the C2 vertebral artery groove, this may be at risk in almost 10% of patients based on CT planning but actual injury is thankfully much rare.

Q. 53 A 29-year-old soldier steps on a landmine on the battlefield. He starts to bleed profusely from a mangled left leg. What is the initial treatment?

A. A battlefield tourniquet should be applied to the limb B. A clean dressing with a military pressure bandage C. A haemostatic dressing should be applied immediately D. Immediate evacuation from the battlefield

Explanation:

Early use of a tourniquet in combat situations has led to a reduction in morbidity from haemorrhagic limb injuries.

Q. 54 A motorcyclist is involved in a road traffic accident (RTA) where he hits a stationary object at approximately 50 m.p.h. At the scene he has a pulse rate of 100 b.p.m. and a blood pressure of 140/80. He is taken to the nearest major trauma centre with a hard collar on a spinal board. On arrival he has a primary survey. His pelvic radiograph shows an anteroposterior compression type II pelvic fracture. Which of the following should be avoided?

A. Application of a pelvic binder if one has not already been applied by paramedics

B. Examination of the perineum

C. Log roll the patient to examine the spine for associated injuries

D. Perform a trauma series CT with contrast as early as possible

Explanation:

Log rolling a patient with a potentially unstable pelvic ring injury is contraindicated. This risks mechanical disturbance to the clot that has formed in the retroperitoneum. This clot will contain a large proportion of the body’s clotting factors and platelets, meaning there is low reserve for the formation of a further clot. A retrograde urethrogram would not be considered in the primary survey; however, an examination of the perineum for open injuries and ecchymosis must be carried out and documented.

Q. 55 What is the maximum acceptable time delay for vascular reconstruction if confirmed vascular impairment exists in a lower limb?

A. 8 hours of cold ischaemia

B. 6 hours of warm ischaemia

C. 3–4 hours of warm ischaemia

D. It depends on coexisting soft/bone tissue injury

Explanation:

Six hours of warm ischaemia represents the amount of time it takes for irreversible ischaemic damage to occur in muscle tissue.

Q.56 which of the following is not an aim in the management of calcaneum fracture?

A. Correct articular incongruity (posterior facet) B. Restore shape- widening/shortening/loss of height C. Correct talar declination angle D. Correct hindfoot varus

Explanation:

Correcting talar declination angle is important for talar neck fracture.

Q 57: chronic TA rupture with gap >2cm. which is not ideal option?

A. V-y advancement B. Tendon graft C. Turn down flap D. Pull tension for stress relaxation of musculotendinous unit

Explanation:

Pull tension using sutures can be used as a method to achieve some length in chronic TA ruptures usually with gap upto 2cm. in cases with gap >2cm adequate length ant be achieved with pull tension.

Q 58: which is the most active metabolite of Vit D?

a. 25 Hydroxycholecalciferol b. 1,25 Hydroxycholecalciferol c. 24,25 Hydroxycholecalciferol d. 7 Dehydrocholesterol

Explanation: hydroxylation in kidneys – 25-hydroxycholecalciferol to 1,25-dihydroxycholecalciferol and 24,25- dihydroxycholecalciferol (Vitamin D deficiency – chronic renal failure)

1,25-dihydroxycholecalciferol (active metabolite)

• Increase calcium and phosphate absorption from gastrointestinal tract

• Increase osteoclastic resorption of bone

Q59: which is not a function of Parathyroid hormone(PTH) ?

a. Increases renal hydroxylation of 25-hydroxycholecalciferol b. Increases resorption of calcium and decreases resorption of phosphate in kidneys c. Increases osteoclastic resorption of bone d. Increases osteoblastic resorption of bone

Explanation:

PTH works on osteoclasts and not osteoblast causing bone resorption.

Q60: which of the following is not a pre requisite of Gait?

a. Stability in stance b. Adequate step c. Foot clearance in stance d. Energy conservation

Explanation:

Foot Clearance is needed in swing phase and not stance phase of gait cycle.

Q61: Ground reaction Ankle Foot Orthosis ( GRAFO)

a. Works on Newton’s third law b. Accelerates knee extension at mid stance c. Compensates for weak Gastrocnemius/soleus d. All of the above

Explanation:

GRAFO works on newtons third law and thus the ‘reaction’ to the sum of body weight and centre of mass acting downwards during stance”. So in case of weak quadriceps it accentuates knee extension in mid stance and compensates for weak plantarflexors.

Q62: on a stress- strain curve

a. Young’s modulus is stress proportional to strain(linear elasticity)

b. Strain is X axis c. Strain is Y axis d. Stress is X axis

Explanation: Young’s modulus of elasticity – stress/strain (N/m2 or Pascal )

Hooke’s law – stress proportional to strain (linear elasticity)

Q63: which of the following is not a determinant of wear of implant?

a. Wear factor b. Sliding distance c. Load applied across bearing surface d. All are independent determinants

Explanation: All of the above are independent determinant of implant wear.

Q 64: which of the following decreases pull out strength of screw?

a. Increasing outer diameter b. Decreasing inner diameter c. Increase thread density d. Decrease thickness of cortex

Explanation: The aim is to maximise the friction between the screw and the bone. So increasing the thickness of cortex will increase the pull out strength

Q. 65: which of the following affects the anterior horn cells?

a. Multiple sclerosis b. Motor neuron disease poliomyelitis c. Hereditary motor and sensory neuropathy d. Syringomyelia

Explanation: Multiple sclerosis- central cord

Hereditary motor sensory neuropathy- peripheral neuropathy

Syringomyelia – intramedullary (central cord)

Motor neuron disease polio- anterior horn cell

Q. 66 Test to check the integrity of anterior oblique band of MUCL is a. Varus stress test b. Milking maneuver in forearm pronation c. Valgus stress test with elbow in full extension d. Valgus stress test with elbow in 20 degree of flexion

Explanation:

The medial ulnar collateral ligament is the primary restraint to valgus instability of the elbow. The MUCL is composed of three components

1. Anterior oblique ligament (primary restraint against valgus stability)

2. Posterior oblique ligament/ posterior bundle

3. Transverse ligament

Apart from MUCL, which has been shown to be the primary stabilizer of the elbow during valgus stress, the radial head and dynamic stabilizers of the elbow such as the flexor-pronator muscle mass provide stability against valgus stress. The anterior oblique ligament itself is composed of two separate bands (anterior and posterior) that provide reciprocal function with the anterior band tight in extension, and the posterior band tight in flexion.

Test to check integrity of MUCL (Anterior oblique ligament):

1. Valgus stress test: flex elbow to 20 to 30 degrees (unlocks the olecranon), externally rotate the humerus, and apply valgus stress

2. Milking maneuver: creates valgus stress by pulling on the patient's thumb with the forearm supinated and elbow flexed at 90 degrees

Q. 67 After hip arthroscopy patient is complaining of numbness over lateral aspect of upper thigh. Which portal placement can cause this?

a. Anterior b. Mid anterior c. Anterolateral d. Posterolateral

Explanation:

Hip arthroscopy portals:

The first portal to be marked is generally the antero-lateral (AL) one, which is placed 1 cm proximal and 1 cm anterior to the tip of the GT.

The most significant risk in placement of anterolateral portal is not the superior gluteal nerve injury, which lies an average of over 4 cm proximal, but rather iatrogenic chondral-labral injury. This is due to the fact that it is the only portal that is placed without direct visualization.

Anterior portal: often it is made after AL portal. Starting point originally described at intersection between superior ridge of greater trochanter and line drawn longitudinally from ASIS. Later it was moved a bit lateral to this line to avoid lateral femoral cutaneous nerve.

The LFCN is at greatest risk with this portal as it can be found within 3 mm. The femoral nerve and the ascending branch of the lateral femoral circumflex artery (LFCA) are both approximately4 cm away. Terminal branches of the LFCA, however, may be as close as 2 mm . Because of the proximity of the LFCN, a superficial skin incision should be made followed by blunt dissection to protect the nerve. As long as the portal is not medial to the line drawn distally

from the ASIS, the femoral artery and vein should not be at risk

The postero-lateral (PL) portal is located in line with the AL mirrored posteriorly to the tip of the GT. (located 1 cm posterior and 1 cm proximal to the posterosuperior tip of the greater trochanter

The sciatic nerve is, on average, 2.9 cm away, while the deep branch of the medial femoral circumflex artery is just over 1 cm away, assuming normal trochanteric anatomy. The risk to the sciatic nerve is increased with hip internal rotation or external rotation.

Mid anterior portal: It is made lateral to anterior portal and medial to the anterolateral portals

These are the standard entry points for central compartment, historically described by Byrd and generally taken as reference for obtaining the others.

Hip arthroscopy portals and structures at risk:

Anterolateral Chondral and labral injury, superior gluteal nerve anterior Lateral femoral cutaneous nerve posterolateral Sciatic nerve, medial femoral circumflex artery

Q. 68 What does arrow indicate in the given MRI picture below?

a. Ligament of Humphrey b. Posterior cruciate ligament c. Bucket handle tear of lateral meniscus d. Anterior cruciate ligament

Explanation:

Meniscofemoral ligaments are straight bands of collagen that attach to the posterior horn of lateral meniscus and lateral part of medial femoral condyle. The anterior meniscofemoral ligament (aMFL) which is leaning anterior to the posterior cruciate ligament (PCL) is also known as ligament of Humphrey, and the posterior meniscofemoral ligament (pMFL) leaning posterior to PCL is known as ligament of Wrisberg. In saggital MRI section both ligaments are seen as ovoid hypointense structures immediately posterior and anterior to the posterior cruciate ligament respectively. On coronal view they are seen as hypointense band parallel to PCL.

Visualization of the meniscofemoral ligament on (a) sagittal and (b) coronal proton density weighted MR images (a) the ligament of Wrisberg (black curved arrow) and the ligament of Humphrey (white arrows) are identified as small, ovoid hypointense structures immediately posterior and anterior to the posterior cruciate ligament, respectively, on sagittal proton density weighted MR images. In (b) the ligament of Wrisberg is visualized as a hypointense band (arrows) on coronal proton density weighted MRI.

Q. 69 True about scapho-lunate dissociation are all except

a. dorsal component of scapholunate interosseous ligament provides the greatest constraint to translation between the scaphoid and lunate b. Watson test is a provocative teat c. SLAC wrist in advance cases d. S-L angle 45 degree in advance cases

Explanation:

Scapho-lunate instability and DISI:

Scapholunate instability is the most frequent pattern of carpal instability and it may occur with or without dorsal intercalated segmental instability (DISI deformity). Scapholunate interosseous ligament (SLIL) has been indicated as the primary stabilizer of the scapholunate (SL) articulation. The SLIL is C-shaped and attaches exclusively along the dorsal, proximal, and volar margins of the articulating surfaces, the dorsal component is the thickest, strongest, and most critical of the SL stabilizers. The dorsoradial carpal ligament (DRCL), the dorsal intercarpal (DIC) ligament, the scaphotrapezial (ST) ligaments, and the radioscaphocapitate (RSC) ligaments have been shown as secondary stabilizers.

The scaphoid, lunate, and triquetrum can be described as an intercalated segment, because no tendons insert upon them. Their motion entirely depends on mechanical signals from their surrounding articulations, checked by an intricate system of intrinsic, or interosseous, and extrinsic carpal ligaments.

Whilst division of the SLIL alone may not lead to any static changes in carpal position, there are significant changes in force transmission and kinematics of the scaphoid and lunate. Division of at least one other secondary restraining ligament is required to show static changes in scaphoid and lunate position.

Clinical features of scapholunate instability:

A history of a hyperextension injury or repetitive trauma such as in chronic crutch walking is common, but a high index of suspicion is required to correctly diagnose scapholunate disruption in the acute setting. Pain is often noted on the radial aspect of the wrist, usually being well localized around the scaphoid and dorsal scapholunate interval. There may be

point tenderness distal to the Lister’s tubercle. Patients may also present with a history of painful clicking, reduced grip strength and giving way with activities.

Watson described a provocative test, known as the scaphoid shift test, in which the wrist is brought from ulnar to radial deviation while the scaphoid tuberosity is stabilised with the examiner’s thumb, a painful click is suggestive of scapholunate instability.

Imaging: On posteroanterior (PA) view and clenched Wrist views if the scapholunate gap is >3 mm (Terry Thomas sign) it suggests scapholunate instability. A cortical ring sign may also be noted in which scaphoid tuberosity is seen in profile due to flexed position of the scaphoid. On the lateral view, the normal scapholunate angle (angle between the scaphoid tangent and the perpendicular to the lunate tangent) is between 30° and 60°. A scapholunate angle of >70° is consistent with scapholunate dissociation.

Radiolunate angle: Normal RL angle is 15 degrees of extension to 15 degrees of flexion. Radiolunate angle >15 degrees of extension or capitolunate angle >15 degrees also suggest DISI. Radio-lunate angle:

Progression of SL instability to SLAC via DISI

Patient gives a history of fall onto an outstretched hand, causing a tear of the SLIL. Patients often have wrist pain but initially there is no positional changes of the scaphoid or lunate on static radiographs. A complete tear of the SLIL with partial extrinsic ligament injury, if left untreated, will lead to abnormal load transfer and kinematics. This is known as dynamic scaphoid instability. Diagnosis requires stress radiographs or motion studies.

Scapholunate dissociation occurs following a tear to one or more of the secondary stabilizers in addition to the SLIL. Lunate rotation occurs independently of the scaphoid, and the lunate falls into a more extended position. Patients will have abnormal static radiographs, and over time, a DISI deformity will develop . Eventually, secondary attritional changes occur to the supporting ligaments, resulting in irreversible postural changes to the scaphoid, capitates and lunate. The abnormal carpal kinematics lead to progressive degenerative changes, known as scapholunate advanced collapse (SLAC).

Dorsal Intercalated Segment Instability (DISI): The most common form of carpal instability. It occurs secondary to disruption of the scapholunate ligamentous complex. The lunate is rotated into extension.

Volar Intercalated Segment Instability (VISI): It occurs secondary to disruption of the lunotriquetral ligamentous complex and results in volar rotation of the lunate and extension of the triquetrum.

Dynamic Instability: It is seen with Incompetent SLIL or complete SLIL tear. Abnormal changes in carpal alignment seen on stress radiographs. Plain radiographs are normal.

Static Instability (Scapholunate dissociation): It occurs following complete SLIL tear along with tear of secondary stabilizer.

Lunotriquetral ligament injury and VISI deformity:

Instability of the lunotriquetral joint caused by rupture of the lunotriquetral ligament and dorsal radiocarpal ligament (aka radiotriquetral ligament). LT ligament injury is less common than SL ligament injury.

VISI Deformity (volar intercalated segment instability) is a type of Carpal Instability Dissociative (CID). It is caused by advanced injury with injury to lunotriquetral ligament, dorsal radiotriquetral ligament and volar radiolunate ligament. VISI may occasionally be seen in uninjured wrists in patients with ligamentous laxity whereas DISI deformity is always pathological.

Clinical features: patient presents with ulnar sides pain that is worse with pronation and ulnar deviation (power grip). Kleinman's shear test and LT shuck test (aka test) may be positive.

X ray: in the wrist lateral view volar flexion of lunate leads to SL angle < 30° (normal is 30-60 degree). Capitolunate zigzag deformity seen with capitolunate angle increase to > 15° (Normally it is 0 degree). Normally no luno-triquetral widening is seen on AP view.

Q. 70 In revision THR there is extensive metaphyseal and diaphyseal bone loss. Intact diaphyseal cortical bone support is < 4 cm. What is the correct statement regarding management

a. It is Paprosky IIIB femoral bone loss, it should be managed with extensively porous coated tapered stem prosthesis with femoral impaction b. It is Paprosky IIIA femoral bone loss, it should be managed with extensively porous coated long stem prosthesis with femoral impaction bone grafting c. It is Paprosky II femoral bone loss, it should be managed with extensively porous coated long stems prosthesis without femoral impaction bone grafting d. It is Paprosky IV femoral bone loss, it should be managed with allograft prosthetic composite (APC)

Explanation:

Paprosky classification of femoral bone: Treatment options according to femoral bone loss

Type description Treatment option

I Minimal metaphyseal bone Revision THR loss

II Extensive metaphyseal bone uncemented extensively loss, minimal diphyseal bone porous-coated long-stem loss prosthesis

IIIA Extensive metaphyseal and uncemented extensively diaphyseal bone loss, ≥ 4 cm porous-coated long-stem intact diaphyseal bone prosthesis

IIIB Extensive metaphyseal and Extensively porus coated diaphyseal bone loss, < 4 cm tapered stems intact diaphyseal bone femoral impaction bone grafting

allograft prosthetic composite (APC)

IV Extensive metaphyseal and femoral impaction bone diaphyseal bone loss, grafting nonsupportive isthmus allograft prosthetic composite (APC)

endoprosthetic replacement (EPR)

Paprosky classification of acetabular bone loss: treatment options

Type 1 Minimal deformity , intact rim porous-coated hemisphere cup or jumbo secured with screws

Type II Superior migration less than 3 cms

Type Superior and medial bone lysis with intact porous-coated hemispherical cup with IIA rim acetabular augments

Type Absent superior rim, superolateral porous-coated hemispherical cup with IIB migration acetabular augments

reconstruction cage with structural bone allograft

Type Localized migration of medial wall porous-coated hemispherical cup with IIC acetabular augments

reconstruction cage with structural bone allograft

Type Superior migration more than 3 cms III

Type Superior and lateral migration, cup up and porous-coated hemispherical cup with IIIA out acetabular augments

reconstruction cage with structural bone allograft

Type Superior and medial migration, cup up and porous-coated hemispherical cup with IIIB in acetabular augments

reconstruction cage with structural bone allograft

Q. 71 Post-operative protocol after Q-snip exposure in TKR is a. Rehabilitation is slow as immobilization is needed b. Rehabilitation is fast compared to midline parapatellar approach

c. Rehabilitation is as same as midline parapatellar approach d. Rehabilitation is slow and some extension lag may occur due to quadriceps weakness

Explanation:

Approaches for TKR

Approach Advantages Disadvantages

Medial Parapatellar Mid line incision, Excellent exposure Lateral patellar Approach even in challenging subluxation cases may jeopardize patellar circulation if lateral release is performed

Lateral Parapatellar Useful for a fixed Medial eversion of Approach valgus deformity, patella is difficult prevents lateral patellar subluxation and preserve blood supply of patella

Midvastus spares the quadriceps -potentially allows less extensile tendon and is carried accelerated rehab due through the belly of to avoiding disruption exposure VMO of extensor difficult in obese mechanism, patients

exposure difficult -Improved patellar with flexion tracking contractures Relative contraindications

-ROM <80 degrees

-obese patient

-hypertrophic arthritis

-previous HTO

Subvastus muscle belly of vastus accelerated rehab due least extensile medialis is lifted off to avoiding disruption intermuscular septum of extensor Relative mechanism, contraindications

-revision TKA -large quadriceps

-previous HTO

-obese patient

-previous parapatellar arthrotomy

Extensile approaches

Advantages Disadvantages

Quadriceps snip snip made at apex of no change in post- not as extensile as quadriceps tendon operative protocol a turndown or obliquely across tendon tibial tubercle at a 45-degree angle into osteotomy vastus lateralis

V-Y turndown allows excellent -extensor lag exposure -may affect allows lengthening of quadriceps quadriceps tendon strength

-knee needs to be immobilized post- operatively

-Rehab is slow

Tibial tubercle -excellent exposure -some surgeons osteotomy immobilize or -avoids extensor lag limit weight- bearing post- operatively

-tibial tubercle avulsion fracture

-non-union

-Rehab is slow

Q. 72 Year female, with no history of trauma, complains of pain on lateral side of the knee. This pain specially increases after running downhill. Physical examination is normal. Probable diagnosis is?

a. Lateral meniscal tear

b. Lateral collateral ligament tear

c. Lateral hamstring strain

d. Iliotibial band syndrome

Explanation:

Iliotibial band syndrome: ITB is a fibrous reinforcement of fascia lata and spans over lateral aspect of the thigh to insert over gerdy’s tubercle on the lateral condyle of the tibia. Athletes requiring repetitive flexion and extension of the knee joint are prone to develop ITB friction syndrome. Repetitive strains of ITB due to flexion and extension of the knee causes compression effect on connective tissues deep to the ITB to produce pain on the lateral side of the knee. It is a common cause of lateral knee pain in runners and cyclist. Tight iliotibial band and weak hip abductors may predispose to ITBFS. Patients with ITB syndrome, complains of lateral knee pain during running and other sports activity. Tenderness can be elicited proximal to lateral joint line.

Noble compression test is specific to ITB friction syndrome. In this test patient is made supine with knee flexed to 90 degrees. Now examiner compresses over the lateral epicondyle proximal

to joint line and patient is asked to extend the knee and hip slowly. In ITBFS, this maneuver will produce pain.

Treatment is mostly conservative with NSAIDS, Ice application and ITB stretching exercises. Local steroid injection may help in non-responders. Surgical resection of posterior half of ITB is required in resistant cases.

Q. 73 Gillquist portal is

a. Anterolateral portal into knee joint

b. Anteromedal portal into knee joint

c. Central portal through patellar tendon

d. Supraclavicular portal

Explanation:

Knee:

Anterolateral: This is the main viewing portal (main diagnostic portal) and first portal to be made. It can view all structures except anterior horn of the lateral meniscus, periphery of posterior horn of the medial meniscus and posterior cruciate ligament (PCL). It is made 1 cm above the lateral joint line and 1 cm lateral to the patellar tendon.

Anteromedial: This is a main working portal. It is made 1 cm above the medial joint line and 1 cm medial to the patellar tendon. It is used for viewing lateral compartment and also for inserting arthroscopic instruments inside the knee.

Posteromedial: It is used for repair of posterior horn meniscal tears, PCL tears and loose body removal from posterior compartment.

Superolateral portal: It is used for viewing the patello-femoral articulation. It is made lateral to quadriceps tendon and 2.5cm above the superolateral pole of patella.

Central transpatellar portal of Gillquist: It is located in midline 1 cm inferior to lower pole of patella. It is used in anterior cruciate ligament (ACL) reconstruction after the graft has been harvested.

Shoulder:

The posterior portal (or “soft-spot” portal) is commonly used as viewing portal and it is the first portal to be made in all the shoulder arthroscopic procedures. It is made 1–2 cm medial and 2 cm inferior to the posterolateral corner of the acromion, by inserting the trocar towards the coracoid process. The axillary nerve inferiorly and the suprascapular artery and nerve medially are potentially at risk while this portal is made.

Anteroinferior Portal (Wolf’s portal): The cutaneous incision is made just laterally and inferiorly to the coracoid process. This portal is normally used for suture anchor placement during anteroinferior capsulolabral repair. The axillary and the musculocutaneous nerve, the axillary artery, and the cephalic vein are potentially at risk.

Anterosuperior Portal : It is located between the coracoid and the acromion. It enters the joint just anterior to the long head of the biceps tendon. This portal is used as outflow and/or retrieval portal during .

Portal of Wilmington: This portal is made under direct visualization 1 cm laterally and 1 cm inferiorly the posterolateral corner of the acromion, passing through the posterolateral part of the cuff. It is used to repair posterior SLAP lesions

5 o’clock portal: The “5 o’clock portal” is used to approach the inferior glenoid rim and facilitate the insertion of the suture anchors at a right angle during Bankart repair.

Neviaser’s portal or the superior portal or supraclavicular fossa portal: it is located in the supraspinatus fossa, between the clavicle (anteriorly), the scapular spine (posteriorly), and the medial border of the acromion (laterally). This portal is sometimes used during suture anchor placement for SLAP repair.

Subacromial space portals:

Lateral portal: This portal is located 2–3 cm inferiorly to the lateral border of the acromion, in the prolongation of the posterior margin of the acromioclavicular (AC) joint.

Posterolateral portal: It is used to introduce the scope during rotator cuff repair. This portal is situated about 2 cm below the lateral margin of the acromion in the prolongation of its posterior edge.

Anterolateral Portal: This is localized 2–3 cm inferiorly to the lateral border of the acromion, in the prolongation of its anterior edge.

Q. 74 In lateral meniscal repair, which structure is at risk?

a. Peroneal nerve

b. Lateral geniculate artery

c. Infrapatellar nerve

d. Lateral collateral ligament

Explanation: Structures at risk in medial meniscal repair: Saphenous nerve and vein.

Structures at risk in lateral meniscal repair: Peroneal nerve.

Popliteal vessels are at risk in repair of posterior horn of lateral meniscus. Lateral geniculate artery is also at risk in lateral meniscal repair as it traverses through posterolateral aspect of capsule at lateral joint line.

Q. 75 . Which antihypertensive should not be given to an athlete without prior permission from sports authority? a. Nitrates b. Diuretics c. Calcium channel blockers d. ACE inhibitors

Explanation:

World Anti Doping Agency has given a list of drugs which are prohibited in athletes

1. Drugs which are prohibited at all times (In and out of competition)

Substances:

Non-approved substances (drugs which have not yet been approved for human use like veterinary drugs)

Anabolic agents

Peptide hormones, growth factors, related substances and mimetics

Beta-2 agonists

Hormone and metabolic modulators

Diuretics and masking agents

Methods:

Manipulation of blood and blood components

Gene doping

Chemical and physical manipulation

2. Drugs which are prohibited in competition

Stimulants

Narcotics

Cannabinoids

Glucocorticoids

3. prohibited in particular sports

Beta blockers

Anabolic agents

1. Anabolic androgenic steroids: (There is a long list of drugs, you can check on: https://www.wadaama.org/sites/default/files/prohibited_list_2018_en.pdf

2. Other anabolic agents

Clenbuterol

Selective androgen receptor modulators (SARMs), e.g. andarine, LGD-4033, ostarine and RAD140;

Tibolone

Zeranol

Zilpaterol

Peptide hormones, growth factors, related substances and mimetics

The following substances, and other substances with similar chemical structure or similar biological effect(s), are prohibited:

1. Erythropoitins and agents affecting erythropoiesis

Erythropietin receptor agonist

Darbepoetins (dEPO)

Erythropoietins (EPO)

EPO based constructs (EPO-Fc, methoxy polyethylene glycol-epoetin beta (CERA))

EPO-mimetic agents and their constructs , e.g. CNTO-530, peginesatide;

Hypoxia-inducible factor activating agents

Argon

Cobalt

Molidustat

Roxadustat (FG-4592)

Xenon

GATA inhibitors

K-11706

TGF Beta inhibitors

Luspatercept

Sotatercept

Innate repair rexeptor agonit

Asialo EPO

Carbamylated EPO (CEPO)

2. Peptide hormones and hormone modulators

Chorionic gondotrophin and luteinizing hormone and their releasing factors

Buserelin

Deslorelin

Gonadorelin

Goserelin

Leuprorelin

Nafarelin

Triptorelin

Corticotrophins and their releasing factors

Corticorelin

Growth hormone and releasing factors

Growth Hormone fragments , e.g. AOD-9604 and hGH 176-191;

Growth Hormone Releasing Hormone (GHRH), e.g. CJC-1293, CJC-1295, sermorelin and tesamorelin;

Growth Hormone Secretagogues (GHS), e.g. ghrelin and ghrelin mimetics, e.g. anamorelin, ipamorelin and tabimorelin;

GH-Releasing Peptides (GHRPs), e.g. alexamorelin, GHRP-1, GHRP-2 (pralmorelin), GHRP-3, GHRP-4, GHRP-5, GHRP-6, and hexarelin;

3. Growth factors

Fibroblast Growth Factors (FGFs)

Hepatocyte Growth Factor (HGF)

Insulin-like Growth Factor-1 (IGF-1), and its analogues;

Mechano Growth Factors (MGFs)

Platelet-Derived Growth Factor (PDGF)

Thymosin-β4 , and its derivatives e.g. TB-500;

Vascular-Endothelial Growth Factor (VEGF)

Additional growth factors or growth factor modulators affecting muscle, tendon or ligament

protein synthesis/degradation, vascularisation, energy utilization, regenerative capacity or fibre type switching.

Beta- 2 Agonists

All selective and non-selective beta-2 agonists, including all optical isomers, are prohibited;

Fenoterol

Formoterol

Higenamine

Indacaterol

Olodaterol

Procaterol

Reproterol

Salbutamol

Salmeterol

Terbutaline

Tulobuterol

Vilanterol

Except:

• Inhaled salbutamol: maximum 1600 micrograms over 24 hours; in divided doses not to exceed 800 micrograms over 12 hours starting from any dose;

• Inhaled formoterol: maximum delivered dose of 54 micrograms over 24 hours;

• Inhaled salmeterol: maximum 200 micrograms over 24 hours.

The presence in urine of salbutamol in excess of 1000 ng/mL or formoterol in excess of 40 ng/mL is not consistent with therapeutic use of the substance and will be considered as an Adverse Analytical Finding (AAF) unless the Athlete proves, through a controlled pharmacokinetic study, that the abnormal result was the consequence of a therapeutic dose (by inhalation) up to the maximum dose indicated above.

Hormone and metabolic modulators

1. Aromatase inhibitors including:

4-androstene-3,6,17 trione (6-oxo)

Aminoglutethimide

Anastrozole

Androsta-1,4,6-triene-3,17-dione (androstatrienedione)

Androsta-3,5-diene-7,17-dione (arimistane)

Exemestane

Formestane

Letrozole

Testolactone

2. Selective estrogen receptor modulators (SERMs):

Raloxifene

Tamoxifen

Toremifene

3. Other anti-estrogenic substances:

Clomifene

Cyclofenil

Fulvestrant

4. Agents modifying myostatin function(s):

Myostatin inhibitors

5. Metabolic modulators:

Activators of the AMP-activated protein kinase (AMPK), eg. AICAR, SR9009;

Peroxisome Proliferator Activated Receptor δ (PPARδ) agonists , e.g. 2-(2-methyl-4-((4-methyl- 2-(4-(trifluoromethyl)phenyl)thiazol-5-yl)methylthio)phenoxy) acetic acid (GW1516, GW501516);

Insulins , and insulin-mimetics;

Meldonium

Trimetazidine

Diuretics and masking agents

The following diuretics and masking agents are prohibited, as are other substances with a similar chemical structure or similar biological effect(s).

Desmopressin; probenecid; plasma expanders, e.g. intravenous administration of albumin, dextran, hydroxyethyl starch and mannitol.

Acetazolamide; amiloride; bumetanide; canrenone; chlortalidone; etacrynic acid; furosemide; indapamide; metolazone; spironolactone; thiazides, e.g. bendroflumethiazide, chlorothiazide and hydrochlorothiazide; triamterene and vaptans, e.g. tolvaptan.

Except:

Drospirenone; pamabrom; and ophthalmic use of carbonic anhydrase inhibitors (e.g. dorzolamide, brinzolamide).

Local administration of felypressin in dental anaesthesia.

The detection in an Athlete’s Sample at all times or In-Competition, as applicable, of any quantity of the following substances subject to threshold limits: formoterol, salbutamol, cathine, ephedrine, methylephedrine and pseudoephedrine, in conjunction with a diuretic or masking agent, will be considered as an Adverse Analytical Finding (AAF) unless the Athlete has an approved Therapeutic Use Exemption (TUE) for that substance in addition to the one granted for the diuretic or masking agent.

Stimulants: There is a long list of drugs, you can check on: https://www.wadaama.org/sites/default/files/prohibited_list_2018_en.pdf

Bupropion, caffeine, nicotine, phenylephrine, phenylpropanolamine, pipradrol, and synephrine: These substances are included in the 2018 Monitoring Program, and are not considered Prohibited Substances.

Cathine: Prohibited when its concentration in urine is greater than 5 micrograms per milliliter.

Ephedrine and methylephedrine: Prohibited when the concentration of either in urine is greater than 10 micrograms per milliliter.

Epinephrine (adrenaline): Not prohibited in local administration, e.g. nasal, ophthalmologic, or co-administration with local anaesthetic agents.

Pseudoephedrine: Prohibited when its concentration in urine is greater than 150 micrograms per milliliter.

Narcotics

Buprenorphine

Dextromoramide

Diamorphine (heroin)

Fentanyl , and its derivatives;

Hydromorphone

Methadone

Morphine

Nicomorphine

Oxycodone

Oxymorphone

Pentazocine

Pethidine

Cannabinoids

Natural cannabinoids, e.g. cannabis, hashish and marijuana,

Synthetic cannabinoids e.g. D9-tetrahydrocannabinol (THC) and other cannabimimetics.

Except: Cannabidiol.

Glucocorticoids

All glucocorticoids are prohibited when administered by oral, intravenous, intramuscular or rectal routes.

Betamethasone

Budesonide

Cortisone

Deflazacort

Dexamethasone

Fluticasone

Hydrocortisone

Methylprednisolone

Prednisolone

Prednisone

Triamcinolone

Beta-blockers

Beta-blockers are prohibited In-Competition only, in the following sports, and also prohibited Out-of-Competition where indicated.

• Archery (WA)

• Automobile (FIA)

• Billiards (all disciplines) (WCBS)

• Darts (WDF)

• Golf (IGF)

• Shooting (ISSF, IPC)*

• Skiing/Snowboarding (FIS) in ski jumping, freestyle aerials/halfpipe and snowboard halfpipe/big air

• Underwater sports (CMAS) in constant-weight apnoea with or without fins, dynamic apnoea with and without fins, free immersion apnoea, Jump Blue apnoea, spearfishing, static apnoea, target shooting and variable weight apnoea.

*Also prohibited Out-of-Competition

Acebutolol

Alprenolol

Atenolol

Betaxolol

Bisoprolol

Bunolol

Carteolol

Carvedilol

Celiprolol

Esmolol

Labetalol

Levobunolol

Metipranolol

Metoprolol

Nadolol

Oxprenolol

Pindolol

Propranolol

Sotalol

Timolol

Q 76 . Which antihypertensive should not be given to a shooter? a. Beta blocker b. ACE inhibitor c. Angiotensin receptor blocker d. Diuretics

Explanation: Beta blockers cause decreased heart rate, reduction in hand tremor, and anxiolysis. These effects may be performance-enhancing in sports in which it is beneficial to have increased steadiness, such as archery, shooting, and billiards.

Q. 77 Force platform is used for

a. To measure spinning velocity of a ball

b. To measure ground reaction force

c. To measure jump force

d. To measure frictional force of sports shoes

Explanation: Force platforms are measuring platforms which are used to measure ground reaction force when a person stands or walk across them. These are used to study biomechanics of gait while walking or running in sports.

Q.78 What is the Injury Severity Score (ISS) for a patient with a penetrating chest wound (Abbreviated Injury Scale, AIS = 4), liver laceration

(AIS = 4), open tibial shaft fracture (AIS = 3), distal radius fracture (AIS = 2), and a large scalp laceration (AIS = 1)?

A. 11 B. 14 C. 32 D. 41

Explanation: The ISS is a system that provides an overall score for patients with multiple injuries. Each injury is as- signed an AIS score from 1 (minor) to 6 (not survivable) and is allocated to one of six body regions (head, face, chest, abdomen, extremities and pelvis, external). Only the single highest AIS score in each body region is used. The three most severely injured body regions have their score squared and added together to produce the ISS score. Scores range from 0 to 75 (52 + 52 + 52), but if anybody region has an AIS of 6 (not survivable) then the ISS is automatically given as 75. Major trauma is usually defined as an ISS of greater than 15. In this case the three most severely injured regions are the chest, abdomen, and extremities, giving an ISS of 42 + 42 + 32 = 41.

Q. 79 True about the fracture shown in the X ray is all except

A. The addition of an external oblique radiograph also can be helpful in assessing the true amount of displacement. B. Milch type II fracture, is the most common type C. For displaced fracture open reduction and fixation is the treatment of the choice D. Posterior dissection can cause osteonecrosis of trochlea

Explanation: This is X ray of fracture lateral condyle of elbow in a child. The Milch type II fracture,which is really a Salter-Harris type II fracture, is the most common type (95%) Weiss et al. classification of lateral condyle fracture has been shown to have prognostic significance. Type I fractures are displaced less than 2 mm, type II fractures are displaced more than 2 mm with an intact cartilaginous hinge, and type III fractures are displaced more than 2 mm without an intact cartilaginous hinge. A. The addition of an internal oblique radiograph can be helpful in assessing the true amount of displacement. Displaced fractures or those with unclear reduction require open reduction and internal fixation. This is done through a lateral approach, taking care to avoid dissection posteriorly that may injure the blood supply to the trochlea, which enters posteriorly, and cause osteonecrosis. Remember: Milch type I fracture, is Salter-Harris type IV epiphyseal fracture. Milch type II fracture, is Salter-Harris type II epiphyseal fracture.

Q. 80 Treatment of choice for well established lateral condyle non union is

A. Observation till maturity B. Open reduction , bone grafting and fixation C. Varus osteotomy D. Watchful neglect

Explanation: Surgery for well-established nonunions of lateral condyle fracture is difficult, and the goals should be to restore a more anatomic alignment of the elbow. Arthrotomy and realignment of the articular surface should be avoided because of the high risk of further elbow stiffness and osteonecrosis; osteotomy generally is a better option. Rigid internal fixation should be used to promote early motion. Tardy ulnar nerve palsy can accompany lateral condyle nonunions and can be treated with ulnar nerve transposition and correction of the cubitus valgus.

Q. 81 Most common complication of fracture shown in the X ray is

A. Non-union

B. Cubitus varus

C. Missed diagnosis

D. Trochlear osteonecrosis

Explanation: The most common complication of medial condyle fracture is failing to make the correct diagnosis. It is often misinterpreted as fracture of medial epicondyle and immobilized. Once the correct diagnosis is made, nonunion is rare and usually results from inadequately stabilized fractures. Complications usually consist of nonunion with resultant cubitus varus, trochlear osteonecrosis, and loss of reduction, and the complication rate can be as high as 33%. Medial condyle fracture in children: Fractures of the medial humeral condyle in children are rare, accounting for 1% of pediatric elbow fractures. They usually occur in slightly older children than fractures of the lateral condyle, around the ages of 3 to 8 years. This injury often is confused with medial epicondylar fracture, which is more common but occurs in older children. Making a radiographic diagnosis can be difficult, especially in younger patients in whom the trochlea has

not yet ossified, which occurs around the age of 8 years. The treatment for displaced fractures is open reduction and internal fixation to ensure joint congruity.

Q. 82 Which is not an indication for RTSA?

A. Massive rotator cuff tear and arthritis with presence of full active shoulder elevation

B. Cuff tear arthropathy

C. Chronic pseudoparalysis with a massive rotator cuff tear and no arthritis

D. Cuff tear arthropathy with pseudoparalysis

Explanation: Indications to RTSA 1. Cuff tear arthropathy 2. Chronic pseudoparalysis with a massive rotator cuff tear and no arthritis Chronic pseudoparalysis with a massive rotator cuff tear develops secondary to loss of the fulcrum when the rotator cuff fails. As the deltoid muscle contracts, the humeral head dynamically translates superiorly, and the deltoid loses its lever to elevate the arm. Patients present without evidence of glenohumeral arthritis and are frustrated with the inability to use their upper extremity .The first line of treatment should be rehabilitation to strengthen the remaining shoulder musculature to discover if the patient can recruit enough accessory muscles to elevate their arm. The RTSA can restore active elevation through restoration of a fulcrum for deltoid function but we believe is indicated only after physical therapy has failed. Contraindications to RTSA: 1. Severely impaired deltoid function, 2. Isolated supraspinatus tear, and the presence of full active shoulder elevation with a massive rotator cuff tear and arthritis RTSA in a patient with glenohumeral arthritis and an isolated supraspinatus (SST) tear. An isolated SST tear with associated arthritis will not produce an unbalanced shoulder. If the shoulder is balanced, an unconstrained total shoulder arthroplasty is appropriate

Q. 83 True about clavicle fracture is

A. Operative fixation is rarely required

B. Figure of eight splinting is treatment of choice for majority of fractures

C. Increasing comminution is an indication for surgery

D. Shortening >2 cms often remodels well and can be treated in a sling

Explanation: Nonoperative treatment consists of the use of a sling for comfort. Figure-of-eight splinting often causes patient discomfort and lacks proven benefit over simple sling support. Canadian Orthopaedic Trauma Society initiated a multicenter prospective randomized trial to compare nonoperative treatment and plate fixation of displaced clavicular fractures. They concluded that operative treatment resulted in improved functional outcomes and lower rates of malunion and nonunion. Fracture-Specific indications for clavicle fracture fixation ■ Displacement > 2 cm ■ Shortening > 2 cm ■ Increasing comminution (>3 fragments) ■ Segmental fractures ■ Open fractures ■ Impending open fractures with soft-tissue compromise ■ Obvious clinical deformity (usually associated with displacement and shortening) ■ Scapular malposition and winging at initial examination

Q. 84 Acceptable reduction of humeral shaft in adult include all except

A. Shortening < 3 cm B. Angulation < 30 degrees C. Rotation< 30 degrees D. No segmental fracture fragment

Explanation: Surgical indications in fracture shaft of humerus Failure to obtain and maintain adequate closed reduction Shortening > 3 cm Rotation > 30 degrees Angulation > 20 degrees ■ Segmental fracture ■ Pathologic fracture ■ Intraarticular extension (shoulder joint, elbow joint)

Q. 85 True about Kiloh-Nevin syndrome is all except

A. No sensory deficit

B. Compression by deep head of pronator teres

C. Compression of PIN

D. Treatment is mostly non surgical

Explanation: Anterior Interosseous Syndrome (Kiloh-Nevin Syndrome) It refers to compression of the AIN branch of Median nerve usually by the deep head of Pronator teres, FDS arch, edge of lacertus fibrosus or Gantzer’s muscle (accessory head of FPL). Typically, these patients have only motor deficits) and they fail to make an “O.K.” sign (Kiloh- Nevin sign, Fig. 8.37B), as flexion of the interphalangeal joint of the thumb (FPL) and the distal interphalangeal joint of the index finger (FDP) is impaired. Parsonage-Turner syndrome is a variant where there are bilateral AIN signs caused by viral induced brachial plexus neuritis. Nonsurgical treatment is same as above but surgical decompression may be needed in nonresponding cases.

Q. 86 A 19 years old boy presents with night pain, local tenderness at proximal leg for 6 months. A biopsy proves an infective pathology. X ray picture is given below. What is not true?

A. Staph aureus is most common cause B. Treatment is curettage and bone grafting under antibiotic cover. C. Elevated blood counts D. Most common site is proximal tibia

Explanation:

X ray given in question shows typical features of Brodies abscess. This is a long standing localized form of osteomyelitis seen in cases where either the virulence of the infective organism is low or the immunity of the host is good, such that the host is able to localize the infection within the metaphysis. Proximal tibia is the most common site of affection and most affected patients are 10–20 years old (slightly older as compared to patients with acute osteomyelitis). Most patients present with mild to moderate localized tenderness and intermittent pain that is worse at night and has persisted for more than 2 weeks. Blood investigations (WBC counts, ESR, CRP) are usually within normal limits or mildly elevated. X-ray shows cavity at metaphyseal-epiphyseal junction with a rim of reactive new bone. Diagnosis can be confirmed by bone biopsy. S. aureus is the most common organism isolated. Treatment is curettage and bone grafting under an antistaphylococcal antibiotic cover.

Q. 87 True about knee ligaments? a. PCL is intracapsular and lined by synovium b. AL bundle of PCL tightens with extension and loosen in flexion c. ALL ligament lies in layer 3 of lateral structures of the knee d. AM bundle of ACL provides rotatory stability against pivot shift.

Explanation:

Anatomy of the knee: Lateral structures:

Layer 1(superficial ITB, Biceps femoris most)

Layer2 Lateral retinaculum

Layer 3 superficial LCL, ALL

Layer 3 deep Capsule, poplitiofibular ligament, popliteus Common peroneal nerve lies between 1st and 2nd layers. Lateral geniculate artery lies between superficial and deep layers of 3rd layer. Medial structures

Layer 1(superficial Sartorius, medial retinaculum most)

Layer 2 Superficial MCL, MPFL, semimembranosus

Layer 3 Deep MCL, coronary ligament, capsule

Hamstring tendons (semitendinosus and gracilis) lies between 1st and 2nd layers.

Functions of the knee ligaments

Ligament Primary Secondary

ACL Resists anterolateral Resists varus displacement at

displacement of the tibia on the full extension femur

PCL Resists posterior tibial Resists varus displacement at displacement, especially at 90 full extension degrees of flexion

LCL Resists varus displacement at Resists posterolateral rotatory 30 degrees of flexion displacement with lesser degrees of flexion

MCL Resists valgus angulation Resist axial rotation along with ACL

PLC Resists posterolateral rotation Posterior displacement of tibia of the tibia on the femur and varus angulation

ACL:

Anatomy : Extrasynovial but intracapsular. ACL consists of 2 bundles. AM and PL

anteromedial (AM)

▪ fibers are parallel in extension

▪ fibers are externally rotated in flexion

▪ tight in both flexion and extension posterolateral (PL)

▪ PL bundle prevents pivot shift

▪ prevents internal tibial rotation with knee near extension

▪ tight in extension, loose in flexion Origin: from lateral femoral condyle. PL bundle originates posterior and distal to AM bundle. Both bundles are attached to medial wall of lateral femoral condyle behind the residents ridge separates the AM and PL bundle on the medial wall of lateral femoral condyle. Insertion: anterior and between the intercondylar eminences of the tibia Blood supply: middle geniculate artery Innervation: tibial nerve Biomechanics: tensile strength: native ACL 2200 N < bone patellar bone tendon (BPTB) 3000N< quadrupled hamstring 4000N.

PCL Anatomy: Extrasynovial and intracapular. It also consists of 2 bundles, AL and PM. Anterolateral bundle: shorter, thicker and stronger (tight in flexion) Posteromedial bundle: longer , thinner and weaker (tight in extension) Ligament of Humphrey (anterior to PCL) and Ligament of Wrisberg (posterior to PCL) are meniscofemoral ligaments which originate from the posterior horn of the lateral meniscus and cross the PCL to get inserted on to the medial femoral condyle. Anterolateral ligament (ALL) : It is present in layer 3rd of lateral structures of the knee. It takes origin from lateral femoral epicondyle and gets inserted on the lateral tibial plateau between gerdy’s tubercle and fibular head. It is attached to lateral meniscal body which divides it into meniscotibial and meniscofemoral portions. Segond fracture is avulsion fracture of ALL (Meniscotibial part) from the anterolateral tibial plateau.

Q. 88 Empty meniscus sign on MRI indicates

A. Bucket handle tear of meniscus B. Discoid meniscus C. Complete radial tear D. Ramp lesion

Explanation:

Know about different MRI signs of meniscus

Double anterior horn sign (double meniscus sign): it indicates flipped meniscal tear

Empty meniscus sign: it indicates complete radial tear

Double PCL sign: indicates bucket handle tear of medial meniscus

Vacuum phenomenon: A vacuum phenomenon is caused by negative pressure within the joint due to the position of the patient, which results in the accumulation of nitrogen gas. It looks like a tear within a discoid meniscus.

Q. 89 A 36 years female patient presents with complaints of posterior knee pain and difficulty in knee bending. She gives a history of knee injury 1 year back. She also finds heaviness in the knee on climbing stairs and prolonged standing. Sagittal view of the MRI is given below. Which is not a correct statement?

A. Treatment is arthroscopic reconstruction of ACL B. Treatment is arthroscopic debulking C. Treatment is close and then open kinetic chain exercises for partial ACL tear D. Treatment is arthroscopic PCL reconstruction

Explanation:

This is a case of Mucoid degeneration of ACL. Treatment is arthroscopic debulking of ACL.

Mucoid degeneration of ACL: Mucoid degeneration (MD) of the anterior cruciate ligament (ACL) is a rare pathological entity with disputed theories of origin. It is characterized by infiltration of mucoid like substance (glycosaminoglycans) interspersed within the substance of ACL causing knee pain and limited motion.

The most common and consistent symptom is knee pain, mostly posterior. Hypothetically, it is caused due to mechanical impingement on the PCL and posterior capsule or by causing bone erosions. Knee pain during flexion is probably due to the tightening of the anteromedial bundle. Knee pain during terminal extension has also been described. Debridement or debulking of ACL relives posterior impingement.

Q. 90 Metaphyseal bucket handle fracture seen in

A. Osteogenesis imperfect B. Battered baby syndrome

C. Trevors disease D. Marfan syndrome

Explanation: Battered baby syndrome:

Child abuse (battered baby) and neglected or nonaccidental injury in children was first described by Kempe et al. in 1962. They had reported that as many as 10–15% of fractures in children younger than 3 years may be a result of child abuse. Owing to such a high incidence, soon “battered baby syndrome (BBS)” became a recognized clinical entity. It is characterized by physical injuries (skin injuries, multiple fractures, subdural hematomas, sexual abuse, etc.) caused by nonaccidental trauma in children. Most cases of BBS are seen before the age of 3 years (more than 50% are younger than 1 year). Accidental fractures are rare before age of 1 year.

Pattern of Injuries

The most common physical injuries are skin injuries (bruises, scrapes and burns).

Fractures, generally multiple, are the second most common presentation. Although no particular fracture pattern, morphology or location is pathognomonic, most common bone fractured is humerus followed by tibia and femur (King et al.). A single transverse fracture in the diaphysis is generally classical in these long bones. Metaphyseal corner fractures and metaphyseal bucket- handle fractures are almost pathognomonic of child abuse, but less commonly seen than diaphyseal fractures. Skull fractures (eggshell fractures, occipital impression fractures, fractures crossing suture lines) and subdural hemorrhages are common in severe cases. Sexual abuse is estimated to occur in approximately 10% of these children.

Q. 91 What is the most common pattern of forearm deformity in the disease shown in X ray?

A. Short ulna with radius head dislocation

B. Short radius with ulnar bowing

C. Short ulna with bowing of radius without head dislocation

D. Radial shortening with ulna dislocation

Explanation:

This is a case of fore arm involvement in multiple osteochondromatois.

The Masada classification for forearm deformity in multiple osteochondromatosis

Type I The main osteochondroma formation is located in the distal portion of the ulna. The ulna is shortened, and there is bowing of the radius. However, the radial head is not dislocated (this is the most common type in 55–61 % of cases).

Type II In addition to ulnar shortening, the radial head is dislocated (22–33 % of patients). In subtype IIA, the radial head is dislocated because of an additional osteochondroma at the proximal metaphysis of the radius. In subtype IIB, whilst there are osteochondromas at the distal ulna, there are none detectable in this region.

Type III The main osteochondroma formation is in the metaphysis of the distal radius, and there is relative shortening of the radius.

Q. 92 Killer turn in PCL reconstruction can be avoided by all except

A. By anterolateral tibial tunnel position B. Tibial inlay technique C. By going 15-20 mm deep from posterior tibial margin while placing tibial jig aimer D. By exiting the tibial tunnel superiorly (upper half of fossa) in PCL fossa

Explanation:

Killer turn has been described in animal and cadaveric studies in context of PCL reconstruction.

The sharp angle where the graft exits the tibial tunnel, is known as “killer turn”, and is believed to be the main reason for residual laxity and graft tear after PCL reconstruction.

• PCL killer turn can be avoided by

➢ By increasing the tibial tunnel angle ( by going 15-20 mm deep from posterior tibial margin while placing tibial jig aimer) to achieve a smaller “killer turn” effect and stronger graft fixation.

➢ By anterolateral tibial tunnel position

Tibial inlay technique

Q. 93 herringbone pattern on histology is seen in

A. Fibrosarcoma B. Ewings sarcoma C. Synovial cell sarcoma D. Malignant fibrous histiocytoma

Explanation:

Fibrosarcoma

Very rare entity; in fact, it has become a diagnosis of exclusion. The characteristic microscopic appearance of fibrosarcoma consists of spindle cells arranged in a herringbone pattern. The

typical presentation is a 5–10-cm, slow-growing, painless mass in the deep soft tissues of the lower extremity in adults aged 30–50 years old

herringbone pattern

Q. 94 Contraindications to HTO is

a. ACL deficiency

b. Varus deformity>10 degree

c. Age >60 years

d. Inflammatory arthritis

Explanation:

High tibia osteotomy (HTO) shifts the mechanical axis from medial to slightly lateral to the midline of the knee to decrease the load of medial compartment.

Indications Contraindications

Age< 60-70 years BMI> 30

Knee flexion should be more than 90 degree

Extension lag<10 degree High smoker

Involvement of only medial tibio-femoral compartment Flexion<90 degree, extension lag> 10 degree

Active compliant patient Inflammatory arthritis

Tibial subluxation> 1cm

Ligamentous instability (Relative contraindication)

Tricompartmental OA

Patellofemoral arthritis

Varus thrust during gait

More than 15 degree varus deformity

Ligament instability is a relative contraindication:

A primary varus is characterized by a primary varus deformity of the lower limb without any ligament instability; this is the best indication for an HTO. Double varus is a condition characterized by the presence of a varus bone deformity accompanied by ligament instability, primarily of the anterior cruciate ligament (ACL). Triple varus is characterized by the association of varus deformity with central ligament insufficiency (anterior and posterior cruciate ligaments) and failure of the posterolateral corner. This condition is usually associated with a varus thrust during gait.

In cases with combined instability and varus malalignment, a combined or staged procedure, including both ligament procedures and HTO, can be considered. So ACL tear is not a contraindication for HTO.

Q.95 Not a feature of osteopetrosis

A. Endobones B. Bone within bone appearance C. Erlenmeyer flask deformity D. Dripping candle wax appearance

Explanation: Osteopetrosis (Marble Bone Disease, Albers-Schönberg Disease)

Osteopetrosis is characterized by defective osteoclastic bone resorption due to defects in their carbonic anhydrase type II proton pump. Defective bone resorption interferes with normal bone remodeling leading to thickened, radiologically dense (white) bones and hence the term, marble bone disease.

Clinical features: Patients present with features of pancytopenia (abnormal bleeding, anemia, infections and failure to thrive) due to encroachment of the marrow cavity by bone overgrowth. Osteomyelitis is common due to decreased immunity. Although bones are thickened but they are defective as there is no remodeling due to lack of osteoclastic activity which makes them suscep- tible to fracture. Bone pain and pathological fractures are thus common. Healing of fractures is generally normal, but abnormal bones are difficult to fix surgically making internal fixation diffi- cult in these patients. Two types have been described, malignant osteopetrosis (autosomal recessive) and benign osteopetrosis (autosomal dominant). Benign form is usually symptom-free and diagnosed incidentally on X-rays prescribed for other reasons.

X-ray features: Thickened, sclerotic (white) bones are characteristic features. Endobones or bone-within-a-bone appearance (radiodense tissues inside the cortices of long bones) is pathognomonic for osteopetrosis. Defective bone remodeling around the knee joint causes typical Erlenmeyer flask deformity. Rugger jersey spine may be a feature as well.

Irregular cortical sclerosis and dripping candle wax appearance are seen in melorheostosis

Q. 96 Trapdoor sign seen in

A. ABC B. GCT C. SBC D. Enchondroma

Explanation: Diagnostic of the SBC is the “fallen fragment/leaf sign” . A small fragment of cortex, sometimes breaks and the piece settles to the most dependent part of the cyst, confirming its empty cystic nature. However, in some cases the broken fragment does not fall as it remains attached to the periosteum but it folds inwards. This is called as “trapdoor sign”.

Q. 97 True about cortical desmoid is all except

A. Posterolateral aspect of the distal femoral metaphysis is most common site B. Biopsy is usually not required C. Treatment is mostly conservative D. MRI shown erosion of cortex with no lytic lesion

Explanation: Cortical desmoid A cortical desmoid is an irregularity in the posteromedial aspect of the distal femoral metaphysis that occurs as a reaction to muscle stress exerted by the adductor magnus, mostly in 10–15 years old boys. Clinical symptoms include soft tissue swelling and pain. Radiographs and MRI reveal erosion of the cortex with a sclerotic base. A biopsy is not warranted. Treatment usually consists of observation only.

Q. 98 Most common organism causing foot deformity given in the picture below is

A. Actinomadura madurae B. Pseudallescheria boydii C. Actinomadura pelletieri D. Madurella mycetomatis

Explanation: Mycetoma (Madura foot) is a chronic granulomatous infection of skin and subcutaneous tissues caused by fungi (eumycetomas) or bacteria (actinomycetes). Bacteria account for about 60% of cases. Common organisms causing Madura foot are as follows: Actinomycetes: • Actinomadura madurae • Actinomadura pelletieri • Streptomyces somaliensis • Nocardia species Eumycetes: • Pseudallescheria boydii • Madurella mycetomatis

Q. 99 Long follow through in balling

a. Increase chances of muscle injury

b. Decreases chances of muscle injury

c. Increases muscle fatigue

d. Has no relation with muscle injury

Explanation: The biomechanics of throwing: Throwing can be divided into four phases 1. Wind-up (500-1000 ms): during this phase shoulder muscles are relatively inactive. 2. Cocking( 500-1000 ms): shoulder move into abduction, extension and external rotation so, at the end of cocking anterior static restraints like anterior –inferior glenohumeral ligament and antero-inferior capsule develop maximum strain. Windup and cocking together account for 80 % of throwing cycle. Maximum external rotation is seen in this phase with peak muscle activation of rotator cuff muscles. Glenohumeral internal rotation defect (GIRD) and internal impingement are associated with this phase. 3. Acceleration (50ms): it is shortest explosive phase and two forces release the ball. First, forceful contraction of internal rotators and second, elastic recoil of tight anterior capsule. Peak muscle activation is seen in elbow extensor(triceps) and then in shoulder internal rotators( pectoralis major and lattisimus dorsi). With release of ball enormous valgus force is applied on the elbow. 4. Deceleration/follow-through: Eccentric contraction of shoulder external rotators, posterior deltoid and scapular stabilizers decelerates the rapid internal rotation and arm motion. It is highest torque phase and considered the most harmful phase. SLAP tear and biceps injuries are associated with this phase. Late cocking and deceleration phases put greatest force on the shoulder.

Q. 100 Correct sequence of repair of structures in an amputated limb is

a. Bone, extensor tendons, flexor tendons, artery, nerve, vein and lastly skin closure.

b. Bone, artery, nerve, vein, extensor tendons, flexor tendons and lastly skin closure.

c. Artery, nerve, vein , bone, extensor tendons, flexor tendons, and lastly skin closure.

d. Artery, nerve, vein, extensor tendons, flexor tendons, bone and lastly skin closure.

Explanation:

100 Reimplantation of amputated limb/digit

Irreversible necrotic changes begin in muscles after 6 hours of ischemia (warm ischemia time), it is preferable to begin the replantation of parts with good muscular bulk (viz. parts proximal to palms and foot) within this time. With cooling (to 4°C), this time may be extended to 12 hours (cold ischemia time). For small parts with less muscle bulk (e.g. digits), warm ischemia time may be extended to 8 hours or more while the cold ischemia time may be as long as 30 hours. If the ischemia time is crossed, replantation is not advisable (especially of muscle rich parts) as there is a risk of revascularization injury that can end up with renal damage due to myoglobinuria, acidosis and hyperkalemia resulting from metabolites released in significant amounts from necrotic muscles.

Transportation of the Amputated Part

The transacted part is first rinsed gently with sterile saline or ringer lactate to remove excess contamination. Then the part is wrapped with sterile gauze, then soaked in sterile ringer lactate or saline and placed in a plastic bag which is then sealed. The bag is placed on ice in an insulated container so that the part is not touching the ice to avoid freezing of the part. Cooling of the amputated part to about 4°C is imperative to prolong the viability of the transacted part.

Sequence of Repair in Reimplantation

Correct sequence of repair is bone, extensor tendons, flexor tendons, artery, nerve, vein and lastly skin closure. So, fractures and dislocations should be dealt with first, but the meticulous vascular repair is given the greatest priority to save the replanted digit or limb.